NASKAR

Tuesday, May 02, 2006

GMAT Prep Tips and feedback from others
1. Following post is by By Twinsplitter at:
http://www.urch.com/forums/just-finished-my-gmat/26097-790-q50-v51-4.html

Haha at first I thought exactly the same thing, especially because I knew that I didn't have any conceptual problems with the stuff on the test and so it must have been a stupid mistake. But then again, I can never manage to make it through an entire quant section without making a stupid mistake, so I wasn't really surprised. What did surprise me tho is that I did better in Verbal than in quant, as quant has always been my strength. But anyways, it's probably a good thing I missed 800 since Stanford takes some sick sort of pride in the fact that they rejected every 800 applicant last year. Still, I do agree that this site will see an 800 soon, especially if Grey ever takes the test again

Disclaimer:I'm sorry if this post is a bit convoluted or too long, I just figured that since many people (especially those new to the site) use these debriefings as a guide, I would put a lot of the great resources from
this site together in one, easy to find spot for them and for everyone else on this site. However, I have tried to make the transition between each section clear (by using boldface), so that if you want to only find my advice on particular sections then it will be easier.
Disclaimer #2: Before I get started on prep strategy, I want to note that I took the LSAT before I took the GMAT, and thus while it may seem like I have not prepped much for CR and RC, it is because I felt that prepping for the LSAT was more than sufficient prep for these two sections. "But I'm not taking the LSAT, so how does this advice help me" you ask? Read on...

Starting Out:
Originally, I had planned to start prepping for my GMAT as soon as I finished the LSAT, which was in early February (in fact I think it was February 12th, exactly 3 months before my GMAT). However, I was exhausted after prepping for the LSAT, and so I decided to wait until I was finished with Winter quarter
finals in mid March.
I had already been frequenting this site for quite some time at that point, and so I knew that starting off with PowerPrep was a great way to know your standing. I took PP1 and the results were: 710 (q47, v40)

Upon looking at the questions I missed, I realized that in quantitative, as is the case for most people, I was mostly hurt by stupid mistakes. However, I had finished that section 15 minutes early, and I knew that if I properly distributed that extra time then I would be in much better shape. Nonetheless, I realized that I was more rusty than I would like in some areas, and so I decided that quant would be the first area I would work on.
In verbal on PP1, I did not get any CR or RC questions wrong. Yup, that's right, I dropped to a v40 SOLELY based on SC. Needless to say, I realized SC was a big weakness of mine that I needed to work on. However, I
think that SC is probably the easiest section to improve your skills in, as a large percentage of it is just memorizing the necessary rules.

My General View on Prep:
I liken prepping for these tests to an athlete preparing for the season. Rather than sort of work each muscle each day, they specifically target one muscle at a time, spending one day doing bicep only workouts, another doing chest workouts, etc. In the same way, I believe that you should target each specific aspect of the test, concentrating on it and really getting in the mode for it, then moving onto the next type of question. However, when you move on, still do 10 questions a day for each previous section you've done. So, for example, if you start off with quant, then two weeks later you focus on SC while doing 10 quant questions a day. Then, when you move on to CR, you do 10 quant questions and 10 SC questions a day, while still keeping your main focus on CR. Note that using this method, you will be spending progressively more time as you get closer to the test, which is probably a good idea anyways.

I think that your knowledge of each subject will become much more solid in this way than it will in the wishy washy way of just doing a little bit of everything all the time. Then, once you start getting closer to the test (i.e. perhaps two weeks before), and after you've targeted each specific section and feel you're sufficiently prepared, then just work on all of them together, the same way an athlete starts doing more general stuff rather than working out once he/she gets closer to game day.

How to Decide Which Aspects to Target First and For How Long:
I believe that, in general, 2 weeks on a specific subject will give you an absolutely solid grasp on it. However, if there are some sections that you feel need more work than others (i.e. if you're strong in CR but weak in SC), then you could spend only one week on the one you're strong at and 3 weeks on your weakness.

In my opinion, it is best to put quant first for two reasons:
1) this site has a lot of great quant questions/resources, and it's easier to utilize them if you're caught up and fresh in quant,
2) Quant is the easiest to keep fresh by doing a few problems a day, so if you put it in the beginning then you still probably won't forget most of it by the time the test comes around.
As far as what to put second, I believe that it is best to put your biggest weakness in verbal second. Why? Because the topics you put near the beginning will be the ones you get the most practice on, since you'll spend 2 weeks targeting them and then will also do 10 questions a day in these topics from
then on.

In other words, here's the prep plan I would recommend to most people:
Quant (2 weeks)
Biggest Verbal Weakness (2-3 weeks)
2nd Biggest Verbal Weakness (2 weeks)
Verbal Strength (1-2 weeks)
All Types of Questions, General Prep, and Practice Tests (2 weeks)
For a total of about 10 weeks.

My own prep was a little different from my recommended, namely in that I didn't prep for CR and RC and thus only targeted two types of questions (quant and SC). However, as I said before, I basically had already targeted CR and RC by preparing for the LSAT.

My prep went as follows (spread over 8 weeks, with two weeks of non-prepping because I had midterms):
Quant (2 weeks)
SC (2 weeks)
General Prep (2 weeks)

While I certainly spent a good amount of time preparing for this test, I didn't do some amazing number of hours (i.e. Ursula's 200 hours). I did about an hour a day on weekdays (not including time spent on TestMagic, which I found to be a great way to procrastinate!), and around 5 hours a day on weekends for a total of about 90 hours. However, if you include time spent on CR and RC for the LSAT, which was about 60 hours, then it totals to 150 hours. I really would have liked to spent more time preparing, but I knew it was impossible, since the University of Chicago is famous for its enjoyment in torturing undergraduates with a ridiculous amount of work (the school's nickname is "where fun comes to die", or "the level of hell daunte forgot").
Note: Any time I found some helpful information on this site, I copy and pasted it into a word document. In general I think this is a good way to keep track of all of the important stuff you see on the site. And, because I did that, now I have a ton of stuff to share with you guys (see resources for each section).

How I Targeted Each Section

Quant:
General Strategy: My prep for quant consisted of three parts (in this order):
1) Going through
Kaplan's Math Workbook, underlining all of the important concepts, making notecards of these concepts, and doing the practice problems to strengthen these concepts.
2) Scouring TestMagic for all of the great resources that I knew it had on quant, and making notecards of the concepts in these resources. (resources listed below).
3) Doing tons of quant problems from my many question sources (sources listed below).

I think the most important thing in quant is knowing how to set up an equation from a word problem. If you can do this, you will get 95% of your quant questions right, guaranteed. How can you get good at this? Through practice.
See my list of sources of quant questions below to see where you can get practice at this.

Probably my biggest weakness starting out in quant was number theory, as I believe is the case for many people. My advice on cracking this type of question would be to do several of these problems, because it's really just the kind of thing that you get better at with practice. There are several great number theory problems on this site, as well as in the sources I'll list below. As you do more of them, you just get a knack for knowing how to go at it.

Here's how I went at number theory problems: First, I would try to use mathematical logic to lead me to the correct answer. Most of the time, this would work, and I would pretty much know what kind of numbers are relevant to the question (i.e. negative fractions, positive intergers). I would then think what would occur with these types of numbers, and this would lead to the answer.

However, if I was unable to crack it using mathematical logic, I would simply try to plug numbers in, using Alakshma's strategy of plugging in (-2, -1, -0.5, 0, 0.5, 1, 2).

Generally, I would come to the answer sooner or later.
For permutations and combinations, I initially spent way too much time on them (hence the plethora of probability and comb/perm links below) but then realized that I need to take everyone's advice and stop paying attention to them so much. All of you would be well advised to do the same! There's much more
important things to spend your time on.

For Statistics, as I said, I just took a class on it last quarter and thus didn't prepare much for it. However, even had I not taken the class, I still feel that most statistics on the GMAT is fairly easy, perhaps because they know
that most people don't really know the concepts in statistics. So just learn the basic concepts (i.e. what a median is, the fact that standard deviation measures spread, how it is calculated--although I doubt you'll actually have to calculate it, it is helpful to understand how to get it when trying to analyze
what it means).

Sources of Quant Questions:
1)
Kaplan's Math Workbook did every problem in the book
2)
Kaplan 2005 (with the CD)
did every problem in the book, as well as all the Problem Solving and Data Sufficiency Tests on the CD. However, I didn't do any of the CAT full length tests, which I'll discuss in practice tests.
3)
Official Guide only did the questions categorized as hard bin by this document.
4) I bought
Kaplan 800 but never ended up having enough time to get to it. However, I've heard great things about it, and would thus recommend getting it.
5)
TestMagic--Quant Section. Like Grey said, if you search all topics started by Nuthan in the DS section, you'll get hundreds of DS questions to practice on. Also, searching posts made by Lego, Grey, and Shaq can be a great way to find the best problems on this site, and it will also show you how the math geniuses approach
problems.
But while we're on the subject of math geniuses--don't be intimidated if they come up with
brilliant solutions you never would have thought of. Many of the quant questions on this site are much more difficult than what you'll see on the real GMAT.

Quant Resources (note--I probably shouldn't even include all the comb/perm stuff on here b/c I know you guys will spend too much time on it then , but I figure if you're going to waste your time on it, might as well have an easier time finding the stuff ):

Must Have:
Great Math Review
Arithmetic and Geometric Progressions
Overview of Combinations
Overview of Permutations
HCF and LCM Stuff -- The beginning is simple, but further down there are some helpful tricks I didn't know about
Venn Diagram
-- check out my post second from the bottom on the first page. Has everything
you need to know about 3 category sets.

Note: For two category sets, it's simply P(AuB) = P(A) + P(B) - P(AnB)

Everything You Need For Prob/Comb/Perm
Compilation Of Prob/Comb/Perm Questions
Compilation of Tough Problems

Also Helpful:
How to do well in quant
Basic info on standard deviation (math reference in Kaplan tells you how to calculate it)
Info on Probability
More Permutations
More Combinations
More Prob/Comb/Perm
A
ngles and Arcs

Sentence Correction
General Strategy: As I said when discussing my PP1 results, I only got around 65% of these right on my first test. By the time of the test, I averaged 1 wrong out of every 100 questions. Here's how I improved so much:
First thing I did was buy
Manhattan GMAT's Sentence Correction Guide. While it's true that, as everyone says,
OG
is the bible for practicing verbal, I would say that this book is the bible for learning the rules of SC. This book is so comprehensive it's amazing. I cannot emphasize enough what an important role this book played in achieving my score. Also, the friend I told you about who got a 750 without studying did actually spend a couple of days studying. The only thing he studied was this book, and as a result his verbal score jumped from 40 on PP1 to 44 on the actual GMAT.

Here's how to utilize the book:
First, go through Manhattan GMAT's SC guide, highlighting every important point (which, in my opinion, is almost every point in the book) and then making notecards out of those points. Memorize them every chance you get (I did this whenever I rode the bus). At the end of each chapter, Manhattan GMAT lists a set of problems in OG which test the concept you learned about in that chapter. Doing the problem set knowing what type of error you're looking for will make you adept at noticing that problem.
Then, once you have gone through every chapter in Manhattan GMAT, and done the corresponding problems in OG, do OG again, starting from problem number 1. This time, you won't know what type of error you'll be
looking for, but you'll have become so good by doing the problem sets that you will start noticing that you've gotten MUCH better at SC.
Regarding doing the problems in OG more than once: I remember someone saying in their debriefing that as long as you're not memorizing the answers in OG, you can do the problems over again, and you can also take PP and have it be an accurate predictor. I couldn't agree more. Read the explanations, but don't memorize them, so that you can practice as much as possible on real GMAT questions.
One final note: I never ended up using the 1000 SC doc because I found that repeating OG was enough, but
if you feel like you're running out of questions, there are several great questions in 1000 SC as well as in the FREE ETS paper tests that I'll provide links to later.

Resources for SC:
Spidey's SC Notes
1000 SC's
Grammar Reference Didn't use it myself, but looks pretty comprehensive for anyone who wants to check it out.

Critical Reasoning
General Strategy: The way I approached CR problems was much different than the way Kaplan (and most books) recommend it. Unlike most people, I don't read the question stem before I read the stimulus. Rather, I read the stimulus first, trying to get a thorough understanding so that regardless of what the question is, I'm ready to attack it. I really think that this helped build my logic skills, so that I was better prepared for any kind of CR question than I would have been if I had a more question-type-specific approach. I feel that had I tried to read the question first, I'd be so focused on trying to find the assumption/implication that I wouldn't understand the argument as a whole intricately enough to analyze the answer choices appropriately. One reason I trusted this approach is that TestMasters, the company known for being the best LSAT prep course, recommends it (and the LSAT is 1/2 CR, so you figure an LSAT prep course would be particularly privy to how to approach the problems). However, each person should take the approach they feel is best!

Recommended Prep Approach:
I think that the reason I was so good at CR is because, as I said above, the LSAT is half CR, and its CR questions are MUCH more difficult than those on the GMAT. They are extremely nitpicky, which helps you become very logical and helps you spot the errors in GMAT arguments in a second. Thus, I would recommend buying the

"Next Ten Actual, Official LSAT PrepTests"
, which contains 500 LSAT CR questions. If you don't want to buy the book but still want a few LSAT questions, download this free LSAT test.
Do those when you're targeting your CR skills, and then start doing the CR in the OG once you start getting closer to the test (just to get used to the GMAT's style of CR). As far as boldfaced questions, I didn't specifically prep for them, although the LSAT contains some questions which are similar (argument structure questions). Like others have said, process of elimination is pretty helpful in the boldface.
For those of you still looking for boldface questions, I heard that akasans has posted a lot of boldfaced CR's on the site.
Resources for CR:
I don't have any, I'm sorry

Reading Comprehension
General Strategy: I don't really have much of a strategy on reading comprehension, I just sort of read it and answer the questions. One thing that I found was that reading on the computer was very easy for me, perhaps because I read articles online all the time. Many people suggest using the economist online, but that costs $$. Instead, check out
McKinsey Quarterly, which will help your ability to read on a computer screen, your knowledge of business examples (if you get a business issue on AWA), and will probably help your career too by making you knowledgeable on several business issues!
One thing which I think helped me a lot on both my RC and
AWA was the fact that I read the editorial section of the Wall Street Journal every morning on the way to school. It does several things for me:

1) Exposes me to complex arguments similar to those in RC and CR
2) Gives me practice reading on topics which I am often unfamiliar with
3) Keeps me informed, so that I have more real life examples to use in

AWA
.
Finally, perhaps my most important piece of advice on RC is to use the RC's that come in that LSAT book (linked above in the CR section) as practice. The LSAT passages are much more complex, and the questions are much more specific, so that you'll be forced to get better at remembering what you read! Use the LSAT book when targeting RC, and then as the test nears, start doing the OG RC's.

Resources for RC-- I'm not sure regarding the quality of any of these because I haven't gone through them, but I did copy good links whenever I saw them in case I needed more practice for RC, so I figured I might as well share
Ten Vocabulary Learning Tips if you feel like not knowing some of the words in the RC's is hindering your
ability to do well (although it's very normal not to know some of them).
More RC Materials
Even More

Analytical Writing Assessment
General Strategy: Spend a couple days before your test thinking of some big fancy words (my words of choice were eludicate, juxtapose, paucity, dearth, and some other ones that I have now forgotten), as well as some real life examples. I have found that if you have 6 real life examples, odds are 3 of them will be moldable (if that's a word) to become relevent to your topic in analysis of an issue. Attached are my

AWA
templates (sorry Stormgal, I only know how to attach things in threads!). They are essentially a hybrid of Erin's, Sybersport's, and several other templates that I have found on this site.
As far as prep for
AWA, I didn't have any. I simply checked a couple topics out, thought about what I'd say for them to get my mind in the writing mode, and that's about it. However, if you would like a book to build your
AWA
, Spiderman recommended this book which seems like it would be helpful because you can see how others approach it and steal some of their arguments!

Resources for AWA:
Erin's Template for Analysis of an Issue
Template for Analysis of an Argument
Sybersport's (who got a 6.0) Advice
Formatting Rules (makes the grader nice to you!)

More General Resources
There's tons of good (free) stuff I found through TestMagic! Here it is:
All 9 Paper Tests
Calculating Paper Test Scores
OG Softcopy
Free Manhattan GMAT CD
Categorization of OG Questions (I know I've linked to it already, but just wanna make sure everyone gets it, it's really helpful!)
Too many resources to name
Answer Grid that Times You
Prep Strategy Site
Prep Strategy Site #2
Prep Strategy Site #3

Timing:
I didn't put much effort into working on timing, mostly because the LSAT is far more time constrained than the GMAT and I was thus able to work very quickly on everything. In other words, by working in high-pressure, time-constrained situations, my timing got better. Thus, I would recommend doing the same, e.g. only giving yourself 15 minutes to do 10 problems rather than 20 minutes. However, only do this once you know the concepts, because otherwise what's the point of going quickly when you don't even know what it is that you're doing quickly!
I think Kaplan's CD is really good for improving timing in Quant...while giving you only 25 minutes for 20 DS questions may seem ridiculous, it sure makes the actual GMAT, with 2 minutes per question, seem much easier.
Practice Tests:
I know it's really helpful to see what people's practice test scores were so that you know where you stand relative to them. Unfortunately, I don't have many practice scores to give you guys! I knew my timing was alright, and so I felt that doing more problems and learning more concepts was more beneficial for me than doing more practice tests. But again, this is pretty unique to my situation because the LSAT had improved my timing so much. For most people, I would recommend taking SEVERAL practice tests.
Anyways, here's the scores on the tests that I did take:
PP1 (before any prep): 710 (q47, v40)
PP2 (after targeting math and SC): 780 (q50, v47)
Kaplan diagnostic--the one in the book: 700 (q49, v45). Don't know how the hell this score breakup comes out to a 700, but I didn't care b/c I knew Kaplan's tests were horrible.

Day Before the Test:
Unlike most people, I didn't go out to dinner or relax the day before my test. Instead, I did several practice
problems, because I noticed that whenever I would take a couple days off from the GMAT, my mind would get out of the GMAT mindset. So, as I've said earlier, do what you feel best fits your own situation!

Hit Rates:
Problem Solving (in the beginning, when I was making stupid mistakes): 90%
Problem Solving (once I got better at preventing stupid mistakes): 97%
Data Sufficiency (when making stupid mistakes): 85%
Data Sufficiency (once I got better at preventing stupid mistakes): 93%
Sentence Correction (first time around, going category by category as assigned by Manhattan GMAT's book): 95%
Sentence Correction (second time around): 98-99%
Critical Reasoning (did about 80 q's from
OG): 95%
Reading Comp: Only ones I did were on the practice tests, and I think my hit rate was around 97%.
Note, however, that pre-LSAT, my CR was around 84% and my RC was 92%, so don't be discouraged if yours are below mine. Also, for SC, remember that my hit rate before Manhattan GMAT was 65% on that one test. So regardless of where you're at, you can get much better by prepping appropriately.

Future Plans
This fall, I am planning on applying to the three top schools which occassionally accept college seniors without work experience: Stanford, Harvard, and Columbia. I am hoping that this GMAT score, my GPA, the fact that my school is pretty good, my extracurriculars, and my internships will be enough to make me part of the 2% of the entering class that these schools accept without work experience. And if not, the University of
Chicago GSB has a special program for undergraduates at the U of C, in which students apply as seniors, and if they are accepted then they get deferred acceptance, working for two years and then returning after that.

GMAT:

The following are the links that I got from:
http://thefamilyguymba.blogspot.com/2005/04/gmat-experince.html

http://www.syvum.com/gmat/http://atheism.about.com/od/logicalfallacies/a/overview.htm http://daveformba.blogspot.com/http://richardbowles.tripod.com/gmat/gmatmenu.htm http://www.test-preps.com/gmat/math_test_sol.php http://novapress.net/gmat/strategies.html http://www.novapress.net/gmat/math.html http://education.kulichki.net/GMAT/math1.html http://www.gmatbuster.org/http://novapress.net/gmat/math.html http://s2s.wharton.upenn.edu/wh-wharton/messages?msg=5423.1 http://geethu.blogspot.com/2004/06/studying-for-gmat.html http://www.crack-gmat.com/gmat-test.htm http://www.testmagic.com/forum/topic.asp?TOPIC_ID=11279 http://www.testmagic.com/forum/forum.asp?FORUM_ID=67 http://www.microedu.com/gmattest/freetest.htm http://mbaleague.blogspot.com/http://mbawire.blogspot.com/2003_06_29_mbawire_archive.html http://www.krysstal.com/binomial.html http://mathforum.org/dr.math/faq/faq.divisibility.html http://www.gmatclub.com/phpbb/viewtopic.php?t=8202 http://gmat.prepedge.com/MBA/GMAT-CAT/questionbank/ http://s2s.wharton.upenn.edu/n/mb/message.asp?webtag=wh-wharton&msg=7849.1 http://education.kulichki.net/GMAT/math.html http://www.800score.com/gmat-home.html http://www.800score.com/guidetc.html http://www.ascenteducation.com/india-mba/iim/cat/questionbank/Archives/April2002/arith1704.shtml http://www.gmattutor.com/tricks2.html http://www.800score.com/guidec8bview1a.html http://www.economist.com/research/StyleGuide/ http://www.sentencecorrection.com/forums/index.php?act=idx http://novapress.net/gmat/data-sufficiency.html http://www.deltacourse.com/


Some additional links:
1. RC resource:
http://www.aldaily.com/

2. Verbal and Analytical question bank:
http://www.urch.com/forums/archive/index.php/f-111.html
3. Score Top:
http://www.scoretop.com/
4. Deltacourse use debankur as a login name and usual password: http://www.deltacourse.com/gmat/gmat-login.asp

Technical Questions For Computer Science Programmers

1. Given a rectangular (cuboidal for the puritans) cake with a rectangular piece removed (any size or orientation), how would you cut the remainder of the cake into two equal halves with one straight cut of a knife ?

2. You're given an array containing both positive and negative integers and required to find the subarray with the largest sum (O(N) a la KBL). Write a routine in C for the above.

3. Given an array of size N in which every number is between 1 and N, determine if there are any duplicates in it. You are allowed to destroy the array if you like. [ I ended up giving about 4 or 5 different solutions for this, each supposedly better than the others ].

4. Write a routine to draw a circle (x ** 2 + y ** 2 = r ** 2) without making use of any floating point computations at all. [ This one had me stuck for quite some time and I first gave a solution that did have floating point computations ].

5. Given only putchar (no sprintf, itoa, etc.) write a routine putlong that prints out an unsigned long in decimal. [ I gave the obvious solution of taking % 10 and / 10, which gives us the decimal value in reverse order. This requires an array since we need to print it out in the correct order. The interviewer wasn't too pleased and asked me to give a solution which didn't need the array ].

6. Give a one-line C expression to test whether a number is a power of 2. [No loops allowed - it's a simple test.]

7. Given an array of characters which form a sentence of words, give an efficient algorithm to reverse the order of the words (not characters) in it.

8. How many points are there on the globe where by walking one mile south, one mile east and one mile north you reach the place where you started.

9. Give a very good method to count the number of ones in a 32 bit number. (caution: looping through testing each bit is not a solution).

10. What are the different ways to say, the value of x can be either a 0 or a 1. Apparently the if then else solution has a jump when written out in assembly. if (x == 0) y=0 else y =x
There is a logical, arithmetic and a datastructure soln to the above problem.
11. Reverse a linked list.
12. Insert in a sorted list
13. In a X's and 0's game (i.e. TIC TAC TOE) if you write a program for this give a gast way to generate the moves by the computer. I mean this should be the fasteset way possible. The answer is that you need to store all possible configurations of the board and the move that is associated with that. Then it boils down to just accessing the right element and getting the corresponding move for it. Do some analysis and do some more optimization in storage since otherwise it becomes infeasible to get the required storage in a DOS machine.

14. I was given two lines of assembly code which found the absolute value of a number stored in two's complement form. I had to recognize what the code was doing. Pretty simple if you know some assembly and some fundaes on number representation.

15. Give a fast way to multiply a number by 7.

16. How would go about finding out where to find a book in a library. (You don't know how exactly the books are organized beforehand).

17. Linked list manipulation.

18. Tradeoff between time spent in testing a product and getting into the market first.

19. What to test for given that there isn't enough time to test everything you want to.

20. First some definitions for this problem: a) An ASCII character is one byte long and the most significant bit in the byte is always '0'. b) A Kanji character is two bytes long. The only characteristic of a Kanji character is that in its first byte the most significant bit is '1'.
Now you are given an array of a characters (both ASCII and Kanji) and, an index into the array. The index points to the start of some character. Now you need to write a function to do a backspace (i.e. delete the character before the given index).

21. Delete an element from a doubly linked list.

22. Write a function to find the depth of a binary tree.

23. Given two strings S1 and S2. Delete from S2 all those characters which occur in S1 also and finally create a clean S2 with the relevant characters deleted.
24. Assuming that locks are the only reason due to which deadlocks can occur in a system. What would be a foolproof method of avoiding deadlocks in the system.
25. Reverse a linked list.
26. Write a small lexical analyzer - interviewer gave tokens. expressions like "a*b" etc.
27. Besides communication cost, what is the other source of inefficiency in RPC?
(answer : context switches, excessive buffer copying). How can you optimise the communication? (ans : communicate through shared memory on same machine, bypassing the kernel _ A Univ. of Wash. thesis)
28. Write a routine that prints out a 2-D array in spiral order!
29. How is the readers-writers problem solved? - using semaphores/ada .. etc.
30. Ways of optimizing symbol table storage in compilers.
31. A walk-through through the symbol table functions, lookup() implementation etc - The interv. was on the Microsoft C team.
32. A version of the "There are three persons X Y Z, one of which always lies"..
etc..
33. There are 3 ants at 3 corners of a triangle, they randomly start moving towards another corner.. what is the probability that they don't collide.
34. Write an efficient algo and C code to shuffle a pack of cards.. this one was a feedback process until we came up with one with no extra storage.
35. The if (x == 0) y = 0 etc..
36. Some more bitwise optimization at assembly level
37. Some general questions on Lex Yacc etc.
38. Given an array t[100] which contains numbers between 1..99. Return the duplicated value. Try both O(n) and O(n-square).
39. Given an array of characters. How would you reverse it. ? How would you reverse it without using indexing in the array.
40. GIven a sequence of characters. How will you convert the lower case characters to upper case characters. ( Try using bit vector - sol given in the C lib -> typec.h)
41. Fundas of RPC.
42. Given a linked list which is sorted. How will u insert in sorted way.
43. Given a linked list How will you reverse it.
44. Tell me the courses you liked and why did you like them.
45. Give an instance in your life in which u were faced with a problem and you tackled it successfully.
46. What is your ideal working environment. ( They usually to hear that u can work in group also.)
47. Why do u think u are smart.
48. Questions on the projects listed on the Resume.
49. Do you want to know any thing about the company.( Try to ask some relevant and interesting question).
50. How long do u want to stay in USA and why?
51. What are your geographical preference?
52. What are your expecctations from the job.
53. Give a good data structure for having n queues ( n not fixed) in a finite memory segment. You can have some data-structure separate for each queue. Try to use at least 90% of the memory space.
54. Do a breadth first traversal of a tree.
55. Write code for reversing a linked list.
56. Write, efficient code for extracting unique elements from a sorted list of array. e.g. (1, 1, 3, 3, 3, 5, 5, 5, 9, 9, 9, 9) -> (1, 3, 5, 9).
57. C++ ( what is virtual function ? what happens if an error occurs in constructor or destructor. Discussion on error handling, templates, unique features of C++. What is different in C++, ( compare with unix).
58. Given a list of numbers ( fixed list) Now given any other list, how can you efficiently find out if there is any element in the second list that is an element of the first list (fixed list).
59. GIven 3 lines of assembly code : find it is doing. IT was to find absolute value.
60. If you are on a boat and you throw out a suitcase, Will the level of water increase.
61. Print an integer using only putchar. Try doing it without using extra storage.
62. write C code for deleting an element from a linked listy traversing a linked list efficient way of elimiating duplicates from an array
63. what are various problems unique to distributed databases
64. declare a void pointer a) void *ptr;
65. make the pointer aligned to a 4 byte boundary in a efficient manner a) assign the pointer to a long number and the number with 11...1100 add 4 to the number
66. what is a far pointer (in DOS)
67. what is a balanced tree
68. given a linked list with the following property node2 is left child of node1, if node2 < node1 els, it is the right child.
O P O A O B O C
How do you convert the above linked list to the form without disturbing the property. Write C code for that.
O P O B / \ / \ / \ O ? O ?
determine where do A and C go
69. Describe the file system layout in the UNIX OS a) describe boot block, super block, inodes and data layout
70. In UNIX, are the files allocated contiguous blocks of data a) no, they might be fragmented how is the fragmented data kept track of a) describe the direct blocks and indirect blocks in UNIX file system
71. Write an efficient C code for 'tr' program. 'tr' has two command line arguments. They both are strings of same length. tr reads an input file, replaces each character in the first string with the corresponding character in the second string. eg. 'tr abc xyz' replaces all 'a's by 'x's, 'b's by 'y's and so on. a) have an array of length 26. put 'x' in array element corr to 'a' put 'y' in array element corr to 'b' put 'z' in array element corr to 'c' put 'd' in array element corr to 'd' put 'e' in array element corr to 'e' and so on.
the code while (!eof) { c = getc(); putc(array[c - 'a']); }
72. what is disk interleaving
73. why is disk interleaving adopted
74. given a new disk, how do you determine which interleaving is the best a) give 1000 read operations with each kind of interleaving determine the best interleaving from the statistics
75. draw the graph with performace on one axis and 'n' on another, where 'n' in the 'n' in n-way disk interleaving. (a tricky question, should be answered carefully)
76. I was a c++ code and was asked to find out the bug in that. The bug was that he declared an object locally in a function and tried to return the pointer to that object. Since the object is local to the function, it no more exists after returning from the function. The pointer, therefore, is invalid outside.
77. A real life problem - A square picture is cut into 16 sqaures and they are shuffled. Write a program to rearrange the 16 squares to get the original big square.

Programming Questions:

This is a summary of the questions I got in 9 in-person interviews with 5 companies and about 10 phone screens 8/02-11/02.
The interviews were pretty evenly split between very large, large, and startup-sized tech companies.

The good news is that interview question repertoire is generally very limited.
The
Programming Interviews Exposed
book covered or helped on probably 60-70% of questions I got. Well worth the
$20.
A) Linked Lists
This is an extremely popular topic. I've had linked lists on every interview.You must be able to produce simple clean linked list implementations quickly.
1. Implement Insert and Delete for
- singly-linked linked list
- sorted linked list
- circular linked list

int Insert(node** head, int data)int Delete(node**
head, int deleteMe)


2. Split a linked list given a pivot value
void Split(node* head, int pivot, node** lt, node** gt)

3. Find if a linked list has a cycle in it. Now do it without marking nodes.

4. Find the middle of a linked list. Now do it while only going through the list once. (same solution as finding cycles)

B) Strings
1. Reverse words in a string (words are separated by one or more spaces). Now do it in-place. By far the most popular string question!

2. Reverse a string

3. Strip whitespace from a string in-place
void StripWhitespace(char* szStr)

4. Remove duplicate chars from a string ("AAA BBB" -> "A B")
int RemoveDups(char* szStr)

5. Find the first non-repeating character in a string:("ABCA" -> B )
int FindFirstUnique(char* szStr)

C) More Advanced Topics:
You may be asked about using Unicode strings. What the interviewer is usually looking for is:
each character will be two bytes (so, for example, char lookup table you may have allocated needs to be expanded from 256 to 256 * 256 = 65536 elements)
- that you would need to use wide char types (wchar_t instead of char)
- that you would need to use wide string functions (like wprintf instead of printf)
Guarding against being passed invalid string pointers or non nul-terminated strings (using walking through a string and catching memory exceptions

D) Binary Trees
Implement the following functions for a binary tree:
- Insert
- PrintInOrder
- PrintPreOrder
- PrintPostOrder

Implement a non-recursive PrintInOrder

E) Arrays
1. You are given an array with integers between 1 and 1,000,000. One integer is in the array twice. How can you determine which one? Can you think of a way to do it using little extra memory.

2. You are given an array with integers between 1 and 1,000,000. One integer is missing. How can you determine which one? Can you think of a way to do it while iterating through the array only once. Is overflow a problem in the solution? Why not?

3. Returns the largest sum of contiguous integers in the arrayExample: if the input is (-10, 2, 3, -2, 0, 5, -15), the largest sum is 8
int GetLargestContiguousSum(int* anData, int len)

4. Implement Shuffle given an array containing a deck of cards and the number of cards. Now make it O(n).
Return the sum two largest integers in an array.
int SumTwoLargest(int* anData, int size)

5. Sum n largest integers in an array of integers where every integer is between 0 and 9int SumNLargest(int* anData, int size, int n)

F) Queues
Implement a Queue class in C++ (which data structure to use internally? why? how to notify of errors?)

G) Other

1. Count the number of set bits in a byte/int32 (7 different solutions)

2. Difference between heap and stack? Write a function to figure out if stack grows up or down.

3. SQL query to select some rows out of a table (only because I had SQL on the resume)

4. Open a file as securely as possible (assume the user is hostile -- list all the nasty things that could happen and checks you would have to do to)

5. Implement a function to return a ratio from a double (ie 0.25 -> 1/4). The function will also take a tolerance so if toleran ce is .01 then FindRatio(.24, .01) -> 1/4int FindRatio(double val, double tolerance,
int& numerator, int& denominator)

6. Write a program that tests whether a floating point number is zero. (Hint: You shouldn't generally use the equality operator == with floating point numbers, since floating point numbers by nature are hard to match exactly. Instead, test whether the number is close to zero.)

public class FloatTest {
public static void main(String[] args) {
float f = 100.0f;
float MAX = 0.001f;
float MIN = -MAX;
System.out.print(String.valueOf(f));
if ((f <> MIN)) {
System.out.println(" is pretty darn close to 0.");
} else {
System.out.println(" is not close to 0.");
}
}
}



F) Puzzles

1. You have 2 supposedly unbreakable light bulbs and a 100-floor building. Using fewest possible drops, determine how much of an impact this type of light bulb can withstand. (i.e. it can withstand a drop from 17th
floor, but breaks from the 18th).Note that the ever-popular binary search will give you a worst case of 50 drops. You should be able to do it with under 20.

2. There are n gas stations positioned along a circular road. Each has a limited supply of gas. You can only drive clockwise around the road. You start with zero gas. Knowing how much gas you need to get from each gas station to the next and how much gas you can get at each station, design an algorithm to find the gas station you need to start at to get all the way around the circle.

3. Out of 10 coins, one weighs less then the others. You have a scale. How can you determine which one weighs less in 3 weighs? Now how would you do it if you didn't know if the odd coin weighs less or more?

4. What is the next line in the following sequence:11121Answer: it's 1211 and the next is 111221

G) Design Questions

1. How would you design a server that has to process a fair number of good number of requests a second. What if you didn't know how many requests you'd be getting? What if requests had different priorities? (I always think of the
Apache design for this question)

2. Design malloc and free. (give up? see
how G++ malloc works or this page for more examples)

3. Design an elevator control system. Don't forget buttons on every floor and supporting multiple elevators. (What objects/methods/properties/how components communicate)

4. Design a chess game (what objects? what methods? which data where? how will it work?)

5. Design a deck of cards class (object/methods/data)

6. How would you design the infrastructure front half of a very large web ecommerce site? what if it was very personalized? (how sessions are handled? where and what you can cache? how to load-balance?)

H) Concurrency

1. Difference between Mutexes and Critical Sections?

2. What are Reentrant Locks? Implement a Reentrant Lock with Mutexes.

3. Implement a thread-safe class that will read/write to/from a buffer

TSBuffer::TSBuffer(int size)
int TSBuffer::Read(char* buff, int max_size)
int TSBuffer::Write(char* buff, int size)

I) Windows-specific Questions

1. What is the IUnknown COM interface?
2. Synchronization primitives available in Windows (see
MSDN documentation)
3. Basic structure of a Win32 program (WinMain, Message processing)

J) Networking

1. Difference between TCP and UDP? When would you want to use one over the other?
2. How would approach guaging performance of webpages/parts on a very large website?

K) Questions you are unlikely to get unless you claim a lot of IP experience

1. How does traceroute work?
2. How does path MTU discovery work?
3. How can one poison a BGP peer?

L) Non-Technical Questions

All of the following are very common. It's best to have canned answers.
1. What do you want to do?
2. Describe your perfect job?
3. How did your interview go? How did you like the group you interviewed with?
4. Rate your C++ proficiency on the scale of 1 to 10.
5. What have you been up to since you were laid off or finished school?
6. Why do you want to work at X?
7. What reservations do you have working at X?
8. Do you like working alone or in a group?
9. Discuss your greatest accomplishment over the last couple years.
10. Discuss one big problem you solved in a work/school project.
11. How do you handle conflict in a group?
12. How much do you want to make? How much did you make at your previous position?

M) Marketing Questions

Questions for Marketing candidates.

1. How would you market a [specific product this company makes] to a [specific population you are familiar with]? (Example: How would you market Word to college students?)
2. How would you expand a [business you are familar with]?

Links:
The best book for tech interviews, in my opinion - "Programming Interviews Exposed"
Joel On Software article about resumes - must read
Joel On Software techInterview section - more questions and answers
Seven Questions Employees Should Ask Before Joining a Startup

Programming Books:

The original and best C reference by the creators of C:

C Programming Language (2nd Edition) by Brian W. Kernighan
and
Dennis M. Ritchie


The original and best C++ reference by the creator of
C++:
The C++ Programming Language (Special 3rd Edition)
by

Bjarne Stroustrup
(creator of C++)

Design Pattern:

1. Java design pattern: http://www.allapplabs.com/java_design_patterns/facade_pattern.htm

Database questions:


DB2 Questions:
Q:
What is SQL?

A:
SQL stands for 'Structured Query Language'.

TOP
Q:
What is SELECT statement?

A:
The SELECT statement lets you select a set of values from a table in a database. The values selected from the database table would depend on the various conditions that are specified in the SQL query.

TOP
Q:
How can you compare a part of the name rather than the entire name?

A:
SELECT * FROM people WHERE empname LIKE '%ab%'Would return a recordset with records consisting empname the sequence 'ab' in empname .

TOP
Q:
What is the INSERT statement?

A:
The INSERT statement lets you insert information into a database.

TOP
Q:
How do you delete a record from a database?

A:
Use the DELETE statement to remove records or any particular column values from a database.

TOP
Q:
How could I get distinct entries from a table?

A:
The SELECT statement in conjunction with DISTINCT lets you select a set of distinct values from a table in a database. The values selected from the database table would of course depend on the various conditions that are specified in the SQL query. ExampleSELECT DISTINCT empname FROM emptable

TOP
Q:
How to get the results of a Query sorted in any order?

A:
You can sort the results and return the sorted results to your program by using ORDER BY keyword thus saving you the pain of carrying out the sorting yourself. The ORDER BY keyword is used for sorting.SELECT empname, age, city FROM emptable ORDER BY empname

TOP
Q:
How can I find the total number of records in a table?

A:
You could use the COUNT keyword , exampleSELECT COUNT(*) FROM emp WHERE age>40

TOP
Q:
What is GROUP BY?

A:
The GROUP BY keywords have been added to SQL because aggregate functions (like SUM) return the aggregate of all column values every time they are called. Without the GROUP BY functionality, finding the sum for each individual group of column values was not possible.

TOP
Q:
What is the difference among "dropping a table", "truncating a table" and "deleting all records" from a table.
A:
Dropping : (Table structure + Data are deleted), Invalidates the dependent objects ,Drops the indexes
Truncating: (Data alone deleted), Performs an automatic commit, Faster than delete
Delete : (Data alone deleted), Doesn’t perform automatic commit

TOP
Q:
What are the Large object types suported by Oracle?

A:
Blob and Clob.

TOP
Q:
Difference between a "where" clause and a "having" clause.
A:
Having clause is used only with group functions whereas Where is not used with.

TOP
Q:
What's the difference between a primary key and a unique key?

A:
Both primary key and unique enforce uniqueness of the column on which they are defined. But by default primary key creates a clustered index on the column, where are unique creates a nonclustered index by default. Another major difference is that, primary key doesn't allow NULLs, but unique key allows one NULL only.

TOP
Q:
What are cursors? Explain different types of cursors. What are the disadvantages of cursors? How can you avoid cursors?
A:
Cursors allow row-by-row prcessing of the resultsets.
Types of cursors: Static, Dynamic, Forward-only, Keyset-driven. See books online for more information.
Disadvantages of cursors: Each time you fetch a row from the cursor, it results in a network roundtrip, where as a normal SELECT query makes only one rowundtrip, however large the resultset is. Cursors are also costly because they require more resources and temporary storage (results in more IO operations). Furthere, there are restrictions on the SELECT statements that can be used with some types of cursors.
Most of the times, set based operations can be used instead of cursors.

TOP
Q:
What are triggers? How to invoke a trigger on demand?
A:
Triggers are special kind of stored procedures that get executed automatically when an INSERT, UPDATE or DELETE operation takes place on a table.
Triggers can't be invoked on demand. They get triggered only when an associated action (INSERT, UPDATE, DELETE) happens on the table on which they are defined.
Triggers are generally used to implement business rules, auditing. Triggers can also be used to extend the referential integrity checks, but wherever possible, use constraints for this purpose, instead of triggers, as constraints are much faster.
Q:
What is a join and explain different types of joins.

A:
Joins are used in queries to explain how different tables are related. Joins also let you select data from a table depending upon data from another table.
Types of joins: INNER JOINs, OUTER JOINs, CROSS JOINs. OUTER JOINs are further classified as LEFT OUTER JOINS, RIGHT OUTER JOINS and FULL OUTER JOINS.

TOP
Q:
What is a self join?

A:
Self join is just like any other join, except that two instances of the same table will be joined in the query.




Oracle Questions:

1. What are the components of Physical database structure of Oracle Database?.
ORACLE database is comprised of three types of files. One or more Data files, two are more Redo Log files, and one or more Control files.
2. What are the components of Logical database structure of ORACLE database?
Tablespaces and the Database's Schema Objects.
3. What is a Tablespace?
A database is divided into Logical Storage Unit called tablespaces. A tablespace is used to grouped related logical structures together.
4. What is SYSTEM tablespace and When is it Created?
Every ORACLE database contains a tablespace named SYSTEM, which is automatically created when the database is created. The SYSTEM tablespace always contains the data dictionary tables for the entire database.
5. Explain the relationship among Database, Tablespace and Data file.
Each databases logically divided into one or more tablespaces One or more data files are explicitly created for each tablespace.
6. What is schema?
A schema is collection of database objects of a User.
7. What are Schema Objects ?Schema objects are the logical structures that directly refer to the database's data. Schema objects include tables, views, sequences, synonyms, indexes, clusters, database triggers, procedures, functions packages anddatabase links.
8. Can objects of the same Schema reside in different tablespaces.?Yes.
9. Can a Tablespace hold objects from different Schemes ?Yes.
10. what is Table ?A table is the basic unit of data storage in an ORACLE database. The tables of a database hold all of the user accessible data. Table data is stored in rows and columns.
11. What is a View ?
A view is a virtual table. Every view has a Query attached to it. (The Query is a SELECT statement that identifies the columns and rows of the table(s) the view uses.)
12. Do View contain Data ?
Views do not contain or store data.
13. Can a View based on another View ?
Yes.
14. What are the advantages of Views ?
Provide an additional level of table security, by restricting access to a predetermined set of rows and columns of a table.Hide data complexity.Simplify commands for the user.Present the data in a different perpecetive from that of the base table.Store complex queries.
15. What is a Sequence ?
A sequence generates a serial list of unique numbers for numerical columns of a database's tables.
16. What is a Synonym ?
A synonym is an alias for a table, view, sequence or program unit.
17. What are the type of Synonyms?
There are two types of Synonyms Private and Public.
18. What is a Private Synonyms ?
A Private Synonyms can be accessed only by the owner.
19. What is a Public Synonyms ?
A Public synonyms can be accessed by any user on the database.
20. What are synonyms used for ?
Synonyms are used to : Mask the real name and owner of an object.Provide public access to an objectProvide location transparency for tables,views or program units of a remote database.Simplify the SQL statements for database users.
21. What is an Index ?
An Index is an optional structure associated with a table to have direct access to rows,which can be created to increase the performance of data retrieval. Index can be created on one or more columns of a table.
22. How are Indexes Update ?
Indexes are automatically maintained and used by ORACLE. Changes to table data are automatically incorporated into all relevant indexes.
23. What are Clusters ?
Clusters are groups of one or more tables physically stores together to share common columns and are often used together.
24. What is cluster Key ?
The related columns of the tables in a cluster is called the Cluster Key.
25. What is Index Cluster ?
A Cluster with an index on the Cluster Key.
26. What is Hash Cluster ?
A row is stored in a hash cluster based on the result of applying a hash function to the row's cluster key value. All rows with the same hash key value are stores together on disk.
27. When can Hash Cluster used ?
Hash clusters are better choice when a table is often queried with equality queries. For such queries the specified cluster key value is hashed. The resulting hash key value points directly to the area on disk that stores the specified rows.
28. What is Database Link ?
A database link is a named object that describes a "path" from one database to another.
29. What are the types of Database Links ?
Private Database Link, Public Database Link & Network Database Link.
30. What is Private Database Link ?
Private database link is created on behalf of a specific user. A private database link can be used only when the owner of the link specifies a global object name in a SQL statement or in the definition of the owner's views or procedures.
31. What is Public Database Link ?
Public database link is created for the special user group PUBLIC. A public database link can be used when any user in the associated database specifies a global object name in a SQL statement or object definition.
32. What is Network Database link ?
Network database link is created and managed by a network domain service. A network database link can be used when any user of any database in the network specifies a global object name in a SQL statement or object definition.
33. What is Data Block ?
ORACLE database's data is stored in data blocks. One data block corresponds to a specific number of bytes of physical database space on disk.
34. How to define Data Block size ?
A data block size is specified for each ORACLE database when the database is created. A database users and allocated free database space in ORACLE datablocks. Block size is specified in INIT.ORA file and cann't be changed latter.
35. What is Row Chaining ?
In Circumstances, all of the data for a row in a table may not be able to fit in the same data block. When this occurs , the data for the row is stored in a chain of data block (one or more) reserved for that segment.
36. What is an Extent ?
An Extent is a specific number of contiguous data blocks, obtained in a single allocation, used to store a specific type of information.
37. What is a Segment ?
A segment is a set of extents allocated for a certain logical structure.
38. What are the different type of Segments ?
Data Segment, Index Segment, Rollback Segment and Temporary Segment.
39. What is a Data Segment ?
Each Non-clustered table has a data segment. All of the table's data is stored in the extents of its data segment. Each cluster has a data segment. The data of every table in the cluster is stored in the cluster's data segment.
40. What is an Index Segment ?
Each Index has an Index segment that stores all of its data.
41. What is Rollback Segment ?
A Database contains one or more Rollback Segments to temporarily store "undo" information.
42. What are the uses of Rollback Segment ?
Rollback Segments are used :To generate read-consistent database information during database recovery to rollback uncommitted transactions for users.
43. What is a Temporary Segment ?
Temporary segments are created by ORACLE when a SQL statement needs a temporary work area to complete execution. When the statement finishes execution, the temporary segment extents are released to the system for future use.
44. What is a Data File ?
Every ORACLE database has one or more physical data files. A database's data files contain all the database data. The data of logical database structures such as tables and indexes is physically stored in the data files allocated for a database.
45. What are the Characteristics of Data Files ?
A data file can be associated with only one database.Once created a data file can't change size.One or more data files form a logical unit of database storage called a tablespace.
46. What is a Redo Log ?
The set of Redo Log files for a database is collectively known as the database's redo log.
47. What is the function of Redo Log ?
The Primary function of the redo log is to record all changes made to data.
48. What is the use of Redo Log Information ?
The Information in a redo log file is used only to recover the database from a system or media failure prevents database data from being written to a database's data files.
49. What does a Control file Contain ?
A Control file records the physical structure of the database. It contains the following information.
Database NameNames and locations of a database's files and redolog files.Time stamp of database creation.
50. What is the use of Control File ?
When an instance of an ORACLE database is started, its control file is used to identify the database and redo log files that must be opened for database operation to proceed. It is also used in database recovery.
51. What is a Data Dictionary ?
The data dictionary of an ORACLE database is a set of tables and views that are used as a read-only reference about the database.It stores information about both the logical and physical structure of the database, the valid users of an ORACLE database, integrity constraints defined for tables in the database and space allocated for a schema object and how much of it is being used.
52. What is an Integrity Constrains ?
An integrity constraint is a declarative way to define a business rule for a column of a table.
53. Can an Integrity Constraint be enforced on a table if some existing table data does not satisfy the constraint ?No.
54. Describe the different type of Integrity Constraints supported by ORACLE ? NOT NULL Constraint - Disallows NULLs in a table's column.UNIQUE Constraint - Disallows duplicate values in a column or set of columns.PRIMARY KEY Constraint - Disallows duplicate values and NULLs in a column or set of columns.FOREIGN KEY Constrain - Require each value in a column or set of columns match a value in a related table's UNIQUE or PRIMARY KEY.CHECK Constraint - Disallows values that do not satisfy the logical expression of the constraint.
55. What is difference between UNIQUE constraint and PRIMARY KEY constraint ?A column defined as UNIQUE can contain NULLs while a column defined as PRIMARY KEY can't contain Nulls.
56. Describe Referential Integrity ?
A rule defined on a column (or set of columns) in one table that allows the insert or update of a row only if the value for the column or set of columns (the dependent value) matches a value in a column of a related table (the referenced value). It also specifies the type of data manipulation allowed on referenced data and the action to be performed on dependent data as a result of any action on referenced data.
57. What are the Referential actions supported by FOREIGN KEY integrity constraint ?
UPDATE and DELETE Restrict - A referential integrity rule that disallows the update or deletion of referenced data.
DELETE Cascade - When a referenced row is deleted all associated dependent rows are deleted.
58. What is self-referential integrity constraint ?If a foreign key reference a parent key of the same table is called self-referential integrity constraint.
59. What are the Limitations of a CHECK Constraint ?
The condition must be a Boolean expression evaluated using the values in the row being inserted or updated and can't contain subqueries, sequence, the SYSDATE,UID,USER or USERENV SQL functions, or the pseudo columns LEVEL or ROWNUM.
60. What is the maximum number of CHECK constraints that can be defined on a column ?No Limit.
SYSTEM ARCHITECTURE :
61. What constitute an ORACLE Instance ?SGA and ORACLE background processes constitute an ORACLE instance. (or) Combination of memory structure and background process.
62. What is SGA ?The System Global Area (SGA) is a shared memory region allocated by ORACLE that contains data and control information for one ORACLE instance.
63. What are the components of SGA ?Database buffers, Redo Log Buffer the Shared Pool and Cursors.
64. What do Database Buffers contain ?
Database buffers store the most recently used blocks of database data. It can also contain modified data that has not yet been permanently written to disk.
65. What do Redo Log Buffers contain ?Redo Log Buffer stores redo entries a log of changes made to the database.
66. What is Shared Pool ?Shared Pool is a portion of the SGA that contains shared memory constructs such as shared SQL areas.
67. What is Shared SQL Area ?A Shared SQL area is required to process every unique SQL statement submitted to a database and contains information such as the parse tree and execution plan for the corresponding statement.
68. What is Cursor ?A Cursor is a handle ( a name or pointer) for the memory associated with a specific statement.
69. What is PGA ?Program Global Area (PGA) is a memory buffer that contains data and control information for a server process.
70. What is User Process ?A user process is created and maintained to execute the software code of an application program. It is a shadow process created automatically to facilitate communication between the user and the server process.
71. What is Server Process ?Server Process handle requests from connected user process. A server process is in charge of communicating with the user process and interacting with ORACLE carry out requests of the associated user process.
72. What are the two types of Server Configurations ?Dedicated Server Configuration and Multi-threaded Server Configuration.
73. What is Dedicated Server Configuration ?In a Dedicated Server Configuration a Server Process handles requests for a Single User Process.
74. What is a Multi-threaded Server Configuration ?In a Multi-threaded Server Configuration many user processes share a group of server process.
75. What is a Parallel Server option in ORACLE ?A configuration for loosely coupled systems where multiple instance share a single physical database is called Parallel Server.
76. Name the ORACLE Background Process ?DBWR - Database Writer.LGWR - Log WriterCKPT - Check PointSMON - System MonitorPMON - Process MonitorARCH - ArchiverRECO - RecoverDnnn - Dispatcher, andLCKn - LockSnnn - Server.
77. What Does DBWR do ?Database writer writes modified blocks from the database buffer cache to the data files.
78.When Does DBWR write to the database ?DBWR writes when more data needs to be read into the SGA and too few database buffers are free. The least recently used data is written to the data files first. DBWR also writes when CheckPoint occurs.
79. What does LGWR do ?Log Writer (LGWR) writes redo log entries generated in the redo log buffer of the SGA to on-line Redo Log File.
80. When does LGWR write to the database ?LGWR writes redo log entries into an on-line redo log file when transactions commit and the log buffer files are full.
81. What is the function of checkpoint(CKPT)?The Checkpoint (CKPT) process is responsible for signaling DBWR at checkpoints and updating all the data files and control files of the database.
82. What are the functions of SMON ?System Monitor (SMON) performs instance recovery at instance start-up. In a multiple instance system (one that uses the Parallel Server), SMON of one instance can also perform instance recovery for other instance that have failed SMON also cleans up temporary segments that are no longer in use and recovers dead transactions skipped during crash and instance recovery because of file-read or off-line errors. These transactions are eventually recovered by SMON when the tablespace or file is brought back on-line SMON also coalesces free extents within the database to make free space contiguous and easier to allocate.
83. What are functions of PMON ?Process Monitor (PMON) performs process recovery when a user process fails PMON is responsible for cleaning up the cache and Freeing resources that the process was using PMON also checks on dispatcher and server processes and restarts them if they have failed.
84. What is the function of ARCH ?Archiver (ARCH) copies the on-line redo log files to archival storage when they are full. ARCH is active only when a database's redo log is used in ARCHIVELOG mode.
85. What is function of RECO ?RECOver (RECO) is used to resolve distributed transactions that are pending due to a network or system failure in a distributed database. At timed intervals,the local RECO attempts to connect to remote databases and automatically complete the commit or rollback of the local portion of any pending distributed transactions.
86. What is the function of Dispatcher (Dnnn) ?Dispatcher (Dnnn) process is responsible for routing requests from connected user processes to available shared server processes and returning the responses back to the appropriate user processes.
87. How many Dispatcher Processes are created ?Atleast one Dispatcher process is created for every communication protocol in use.
88. What is the function of Lock (LCKn) Process ?Lock (LCKn) are used for inter-instance locking when the ORACLE Parallel Server option is used.
89. What is the maximum number of Lock Processes used ?Though a single LCK process is sufficient for most Parallel Server systemsupto Ten Locks (LCK0,....LCK9) are used for inter-instance locking.
DATA ACCESS
90. Define Transaction ?A Transaction is a logical unit of work that comprises one or more SQL statements executed by a single user.
91. When does a Transaction end ?When it is committed or Rollbacked.
92. What does COMMIT do ?COMMIT makes permanent the changes resulting from all SQL statements in the transaction. The changes made by the SQL statements of a transaction become visible to other user sessions transactions that start only after transaction is committed.
93. What does ROLLBACK do ?ROLLBACK retracts any of the changes resulting from the SQL statements in the transaction.
94. What is SAVE POINT ?For long transactions that contain many SQL statements, intermediate markers or savepoints can be declared which can be used to divide a transaction into smaller parts. This allows the option of later rolling back all work performed from the current point in the transaction to a declared savepoint within the transaction.
95. What is Read-Only Transaction ?A Read-Only transaction ensures that the results of each query executed in the transaction are consistant with respect to the same point in time.
96. What is the function of Optimizer ?
The goal of the optimizer is to choose the most efficient way to execute a SQL statement.
97. What is Execution Plan ?The combinations of the steps the optimizer chooses to execute a statement is called an execution plan.
98. What are the different approaches used by Optimizer in choosing an execution plan ?Rule-based and Cost-based.
99. What are the factors that affect OPTIMIZER in choosing an Optimization approach ?The OPTIMIZER_MODE initialization parameter Statistics in the Data Dictionary the OPTIMIZER_GOAL parameter of the ALTER SESSION command hints in the statement.
100. What are the values that can be specified for OPTIMIZER MODE Parameter ?COST and RULE.
101. Will the Optimizer always use COST-based approach if OPTIMIZER_MODE is set to "Cost'?
Presence of statistics in the data dictionary for atleast one of the tables accessed by the SQL statements is necessary for the OPTIMIZER to use COST-based approach. Otherwise OPTIMIZER chooses RULE-based approach.
102. What is the effect of setting the value of OPTIMIZER_MODE to 'RULE' ?
This value causes the optimizer to choose the rule_based approach for all SQL statements issued to the instance regardless of the presence of statistics.
103. What are the values that can be specified for OPTIMIZER_GOAL parameter of the ALTER SESSION Command ?
CHOOSE,ALL_ROWS,FIRST_ROWS and RULE.
104. What is the effect of setting the value "CHOOSE" for OPTIMIZER_GOAL, parameter of the ALTER SESSION Command ?The Optimizer chooses Cost_based approach and optimizes with the goal of best throughput if statistics for atleast one of the tables accessed by the SQL statement exist in the data dictionary. Otherwise the OPTIMIZER chooses RULE_based approach.
105. What is the effect of setting the value "ALL_ROWS" for OPTIMIZER_GOAL parameter of the ALTER SESSION command ?This value causes the optimizer to the cost-based approach for all SQL statements in the session regardless of the presence of statistics and to optimize with a goal of best throughput.
106. What is the effect of setting the value 'FIRST_ROWS' for OPTIMIZER_GOAL parameter of the ALTER SESSION command ?This value causes the optimizer to use the cost-based approach for all SQL statements in the session regardless of the presence of statistics and to optimize with a goal of best response time.
107. What is the effect of setting the 'RULE' for OPTIMIER_GOAL parameter of the ALTER SESSION Command ?This value causes the optimizer to choose the rule-based approach for all SQL statements in a session regardless of the presence of statistics.
108. What is RULE-based approach to optimization ?Choosing an executing planbased on the access paths available and the ranks of these access paths.
109. What is COST-based approach to optimization ?Considering available access paths and determining the most efficient execution plan based on statistics in the data dictionary for the tables accessed by the statement and their associated clusters and indexes.
PROGRAMMATIC CONSTRUCTS
110. What are the different types of PL/SQL program units that can be defined and stored in ORACLE database ?
Procedures and Functions,Packages and Database Triggers.
111. What is a Procedure ?A Procedure consist of a set of SQL and PL/SQL statements that are grouped together as a unit to solve a specific problem or perform a set of related tasks.
112. What is difference between Procedures and Functions ?A Function returns a value to the caller where as a Procedure does not.
113. What is a Package ?A Package is a collection of related procedures, functions, variables and other package constructs together as a unit in the database.
114. What are the advantages of having a Package ?Increased functionality (for example,global package variables can be declared and used by any proecdure in the package) and performance (for example all objects of the package are parsed compiled, and loaded into memory once)
115. What is Database Trigger ?A Database Trigger is procedure (set of SQL and PL/SQL statements) that is automatically executed as a result of an insert in,update to, or delete from a table.
116. What are the uses of Database Trigger ?Database triggers can be used to automatic data generation, audit data modifications, enforce complex Integrity constraints, and customize complex security authorizations.
117. What are the differences between Database Trigger and Integrity constraints ?A declarative integrity constraint is a statement about the database that is always true. A constraint applies to existing data in the table and any statement that manipulates the table.
A trigger does not apply to data loaded before the definition of the trigger, therefore, it does not guarantee all data in a table conforms to the rules established by an associated trigger.
A trigger can be used to enforce transitional constraints where as a declarative integrity constraint cannot be used.
DATABASE SECURITY
118. What are Roles ?Roles are named groups of related privileges that are granted to users or other roles.
119. What are the use of Roles ?REDUCED GRANTING OF PRIVILEGES - Rather than explicitly granting the same set of privileges to many users a database administrator can grant the privileges for a group of related users granted to a role and then grant only the role to each member of the group.
DYNAMIC PRIVILEGE MANAGEMENT - When the privileges of a group must change, only the privileges of the role need to be modified. The security domains of all users granted the group's role automatically reflect the changes made to the role.
SELECTIVE AVAILABILITY OF PRIVILEGES - The roles granted to a user can be selectively enable (available for use) or disabled (not available for use). This allows specific control of a user's privileges in any given situation.
APPLICATION AWARENESS - A database application can be designed to automatically enable and disable selective roles when a user attempts to use the application.
120. How to prevent unauthorized use of privileges granted to a Role ?By creating a Role with a password.
121. What is default tablespace ?The Tablespace to contain schema objects created without specifying a tablespace name.
122. What is Tablespace Quota ?The collective amount of disk space available to the objects in a schema on a particular tablespace.
123. What is a profile ?Each database user is assigned a Profile that specifies limitations on various system resources available to the user.
124. What are the system resources that can be controlled through Profile ?The number of concurrent sessions the user can establish the CPU processing time available to the user's session the CPU processing time available to a single call to ORACLE made by a SQL statement the amount of logical I/O available to the user's session the amout of logical I/O available to a single call to ORACLE made by a SQL statement the allowed amount of idle time for the user's session the allowed amount of connect time for the user's session.
125. What is Auditing ?Monitoring of user access to aid in the investigation of database use.
126. What are the different Levels of Auditing ?Statement Auditing, Privilege Auditing and Object Auditing.
127. What is Statement Auditing ?Statement auditing is the auditing of the powerful system privileges without regard to specifically named objects.
128. What is Privilege Auditing ?Privilege auditing is the auditing of the use of powerful system privileges without regard to specifically named objects.
129. What is Object Auditing ?Object auditing is the auditing of accesses to specific schema objects without regard to user.

DISTRIBUTED PROCESSING AND DISTRIBUTED DATABASES
130. What is Distributed database ?A distributed database is a network of databases managed by multiple database servers that appears to a user as single logical database. The data of all databases in the distributed database can be simultaneously accessed and modified.
131. What is Two-Phase Commit ?Two-phase commit is mechanism that guarantees a distributed transaction either commits on all involved nodes or rolls back on all involved nodes to maintain data consistency across the global distributed database. It has two phase, a Prepare Phase and a Commit Phase.
132. Describe two phases of Two-phase commit ?Prepare phase - The global coordinator (initiating node) ask a participants to prepare (to promise to commit or rollback the transaction, even if there is a failure)
Commit - Phase - If all participants respond to the coordinator that they are prepared, the coordinator asks all nodes to commit the transaction, if all participants cannot prepare, the coordinator asks all nodes to roll back the transaction.
133. What is the mechanism provided by ORACLE for table replication ?Snapshots and SNAPSHOT LOGs
134. What is a SNAPSHOT ?Snapshots are read-only copies of a master table located on a remote node which is periodically refreshed to reflect changes made to the master table.
135. What is a SNAPSHOT LOG ?A snapshot log is a table in the master database that is associated with the master table. ORACLE uses a snapshot log to track the rows that have been updated in the master table. Snapshot logs are used in updating the snapshots based on the master table.
136. What is a SQL * NET?SQL *NET is ORACLE's mechanism for interfacing with the communication protocols used by the networks that facilitate distributed processing and distributed databases. It is used in Clint-Server and Server-Server communications.
DATABASE OPERATION, BACKUP AND RECOVERY
137. What are the steps involved in Database Startup ?Start an instance, Mount the Database and Open the Database.
138. What are the steps involved in Database Shutdown ?Close the Database, Dismount the Database and Shutdown the Instance.
139. What is Restricted Mode of Instance Startup ?An instance can be started in (or later altered to be in) restricted mode so that when the database is open connections are limited only to those whose user accounts have been granted the RESTRICTED SESSION system privilege.
140. What are the different modes of mounting a Database with the Parallel Server ?
Exclusive Mode If the first instance that mounts a database does so in exclusive mode, only that Instance can mount the database.
Parallel Mode If the first instance that mounts a database is started in parallel mode, other instances that are started in parallel mode can also mount the database.
141. What is Full Backup ?A full backup is an operating system backup of all data files, on-line redo log files and control file that constitute ORACLE database and the parameter.
142. Can Full Backup be performed when the database is open ?No.
143. What is Partial Backup ?A Partial Backup is any operating system backup short of a full backup, taken while the database is open or shut down.
144.WhatisOn-lineRedoLog?The On-line Redo Log is a set of tow or more on-line redo files that record all committed changes made to the database. Whenever a transaction is committed, the corresponding redo entries temporarily stores in redo log buffers of the SGA are written to an on-line redo log file by the background process LGWR. The on-line redo log files are used in cyclical fashion.
145. What is Mirrored on-line Redo Log ?A mirrored on-line redo log consists of copies of on-line redo log files physically located on separate disks, changes made to one member of the group are made to all members.
146. What is Archived Redo Log ?Archived Redo Log consists of Redo Log files that have archived before being reused.
147. What are the advantages of operating a database in ARCHIVELOG mode over operating it in NO ARCHIVELOG mode ?Complete database recovery from disk failure is possible only in ARCHIVELOG mode.Online database backup is possible only in ARCHIVELOG mode.
148. What is Log Switch ?The point at which ORACLE ends writing to one online redo log file and begins writing to another is called a log switch.
149. What are the steps involved in Instance Recovery ?R_olling forward to recover data that has not been recorded in data files, yet has been recorded in the on-line redo log, including the contents of rollback segments.
Rolling back transactions that have been explicitly rolled back or have not been committed as indicated by the rollback segments regenerated in step a. Releasing any resources (locks) held by transactions in process at the time of the failure.
Resolving any pending distributed transactions undergoing a two-phase commit at the time of the instance failure.
Data Base Administration
Introduction to DBA
1. What is a Database instance ? Explain
A database instance (Server) is a set of memory structure and background processes that access a set of database files.
The process can be shared by all users.
The memory structure that are used to store most queried data from database. This helps up to improve database performance by decreasing the amount of I/O performed against data file.
2. What is Parallel Server ?
Multiple instances accessing the same database (Only In Multi-CPU environments)
3. What is a Schema ?
The set of objects owned by user account is called the schema.
4. What is an Index ? How it is implemented in Oracle Database ?
An index is a database structure used by the server to have direct access of a row in a table.
An index is automatically created when a unique of primary key constraint clause is specified in create table comman (Ver 7.0)
5. What is clusters ?
Group of tables physically stored together because they share common columns and are often used together is called Cluster.
6. What is a cluster Key ?
The related columns of the tables are called the cluster key. The cluster key is indexed using a cluster index and its value is stored only once for multiple tables in the cluster.
7. What are the basic element of Base configuration of an oracle Database ?
It consists ofone or more data files.one or more control files.two or more redo log files.The Database containsmultiple users/schemasone or more rollback segmentsone or more tablespacesData dictionary tablesUser objects (table,indexes,views etc.,)The server that access the database consists ofSGA (Database buffer, Dictionary Cache Buffers, Redo log buffers, Shared SQL pool)SMON (System MONito)PMON (Process MONitor)LGWR (LoG Write)DBWR (Data Base Write)ARCH (ARCHiver)CKPT (Check Point)RECODispatcherUser Process with associated PGS
8. What is a deadlock ? Explain .
Two processes wating to update the rows of a table which are locked by the other process then deadlock arises.
In a database environment this will often happen because of not issuing proper row lock commands. Poor design of front-end application may cause this situation and the performance of server will reduce drastically.
These locks will be released automatically when a commit/rollback operation performed or any one of this processes being killed externally.
MEMORY MANAGEMENT
9. What is SGA ? How it is different from Ver 6.0 and Ver 7.0 ?
The System Global Area in a Oracle database is the area in memory to facilitates the transfer of information between users. It holds the most recently requested structural information between users. It holds the most recently requested structural information about the database.
The structure is Database buffers, Dictionary cache, Redo Log Buffer and Shared SQL pool (ver 7.0 only) area.
10. What is a Shared SQL pool ?
The data dictionary cache is stored in an area in SGA called the Shared SQL Pool. This will allow sharing of parsed SQL statements among concurrent users.
11. What is mean by Program Global Area (PGA) ?
It is area in memory that is used by a Single Oracle User Process.
12. What is a data segment ?
Data segment are the physical areas within a database block in which the data associated with tables and clusters are stored.
13. What are the factors causing the reparsing of SQL statements in SGA?
Due to insufficient Shared SQL pool size.
Monitor the ratio of the reloads takes place while executing SQL statements. If the ratio is greater than 1 then increase the SHARED_POOL_SIZE.
LOGICAL & PHYSICAL ARCHITECTURE OF DATABASE.
14. What is Database Buffers ?
Database buffers are cache in the SGA used to hold the data blocks that are read from the data segments in the database such as tables, indexes and clusters DB_BLOCK_BUFFERS parameter in INIT.ORA decides the size.
15. What is dictionary cache ?
Dictionary cache is information about the databse objects stored in a data dictionary table.
16. What is meant by recursive hints ?
Number of times processes repeatedly query the dictionary table is called recursive hints. It is due to the data dictionary cache is too small. By increasing the SHARED_POOL_SIZE parameter we can optimize the size of Data Dictionary Cache.
17. What is meant by redo log buffer ?
Change made to entries are written to the on-line redo log files. So that they can be used in roll forward operations during database recoveries. Before writing them into the redo log files, they will first brought to redo log buffers in SGA and LGWR will write into files frequently.LOG_BUFFER parameter will decide the size.
18. How will you swap objects into a different table space for an existing database ?
Export the user
Perform import using the command imp system/manager file=export.dmp indexfile=newrite.sql. This will create all definitions into newfile.sql.
Drop necessary objects.
Run the script newfile.sql after altering the tablespaces.
Import from the backup for the necessary objects.
19. List the Optional Flexible Architecture (OFA) of Oracle database ? or How can we organise the tablespaces in Oracle database to have maximum performance ?
SYSTEM - Data dictionary tables.DATA - Standard operational tables.DATA2- Static tables used for standard operationsINDEXES - Indexes for Standard operational tables.INDEXES1 - Indexes of static tables used for standard operations.TOOLS - Tools table.TOOLS1 - Indexes for tools table.RBS - Standard Operations Rollback Segments,RBS1,RBS2 - Additional/Special Rollback segments.TEMP - Temporary purpose tablespaceTEMP_USER - Temporary tablespace for users.USERS - User tablespace.
20. How will you force database to use particular rollback segment ?
SET TRANSACTION USE ROLLBACK SEGMENT rbs_name.
21. What is meant by free extent ?
A free extent is a collection of continuous free blocks in tablespace. When a segment is dropped its extents are reallocated and are marked as free.
22. How free extents are managed in Ver 6.0 and Ver 7.0 ?
Free extents cannot be merged together in Ver 6.0.Free extents are periodically coalesces with the neighboring free extent inVer 7.0
23.Which parameter in Storage clause will reduce no. of rows per block?
PCTFREE parameter
Row size also reduces no of rows per block.
24. What is the significance of having storage clause ?
We can plan the storage for a table as how much initial extents are required, how much can be extended next, how much % should leave free for managing row updations etc.,
25. How does Space allocation table place within a block ?
Each block contains entries as followsFixied block headerVariable block headerRow Header,row date (multiple rows may exists)PCTEREE (% of free space for row updation in future)
26. What is the role of PCTFREE parameter is Storage clause ?
This is used to reserve certain amount of space in a block for expansion of rows.
27. What is the OPTIMAL parameter ?
It is used to set the optimal length of a rollback segment.
28. What is the functionality of SYSTEM table space ?
To manage the database level transactions such as modifications of the data dictionary table that record information about the free space usage.
29. How will you create multiple rollback segments in a database ?
Create a database which implicitly creates a SYSTEM Rollback Segment in a SYSTEM tablespace.
Create a Second Rollback Segment name R0 in the SYSTEM tablespace.
Make new rollback segment available (After shutdown, modify init.ora file and Start database)
Create other tablespaces (RBS) for rollback segments.
Deactivate Rollback Segment R0 and activate the newly created rollback segments.
30. How the space utilisation takes place within rollback segments ?
It will try to fit the transaction in a cyclic fashion to all existing extents. Once it found an extent is in use then it forced to acquire a new extent (No. of extents is based on the optimal size)
31. Why query fails sometimes ?
Rollback segment dynamically extent to handle larger transactions entry loads.
A single transaction may wipeout all avaliable free space in the Rollback Segment Tablespace. This prevents other user using Rollback segments.
32. How will you monitor the space allocation ?
By quering DBA_SEGMENT table/view.
33. How will you monitor rollback segment status ?
Querying the DBA_ROLLBACK_SEGS viewIN USE - Rollback Segment is on-line.AVAILABLE - Rollback Segment available but not on-line.OFF-LINE - Rollback Segment off-lineINVALID - Rollback Segment Dropped.NEEDS RECOVERY - Contains data but need recovery or corupted.PARTLY AVAILABLE - Contains data from an unresolved transaction involving a distributed database.
34. List the sequence of events when a large transaction that exceeds beyond its optimal value when an entry wraps and causes the rollback segment to expand into another extend.
Transaction Begins.
An entry is made in the RES header for new transactions entry
Transaction acquires blocks in an extent of RBS
The entry attempts to wrap into second extent. None is available, so that the RBS must extent.
The RBS checks to see if it is part of its OPTIMAL size.RBS chooses its oldest inactive segment.Oldest inactive segment is eliminated.RBS extentsThe Data dictionary table for space management are updated.Transaction Completes.
35. How can we plan storage for very large tables ?
Limit the number of extents in the tableSeparate Table from its indexes.Allocate Sufficient temporary storage.
36. How will you estimate the space required by a non-clustered tables?
Calculate the total header sizeCalculate the available dataspace per data blockCalculate the combined column lengths of the average rowCalculate the total average row size.Calculate the average number rows that can fit in a blockCalculate the number of blocks and bytes required for the table.
After arriving the calculation, add 10 % additional space to calculate the initial extent size for a working table.
37. It is possible to use raw devices as data files and what is the advantages over file. system files ?
Yes.
The advantages over file system files.
I/O will be improved because Oracle is bye-passing the kernnel which writing into disk.Disk Corruption will be very less.
38. What is a Control file ?
Database's overall physical architecture is maintained in a file called control file. It will be used to maintain internal consistency and guide recovery operations. Multiple copies of control files are advisable.
39. How to implement the multiple control files for an existing database ?
Shutdown the databseCopy one of the existing control file to new locationEdit Config ora file by adding new control file.nameRestart the database.
40. What is meant by Redo Log file mirrorring ? How it can be achieved?
Process of having a copy of redo log files is called mirroring.
This can be achieved by creating group of log files together, so that LGWR will automatically writes them to all the members of the current on-line redo log group. If any one group fails then database automatically switch over to next group. It degrades performance.
41. What is advantage of having disk shadowing/ Mirroring ?
Shadow set of disks save as a backup in the event of disk failure. In most Operating System if any disk failure occurs it automatically switchover to place of failed disk.
Improved performance because most OS support volume shadowing can direct file I/O request to use the shadow set of files instead of the main set of files. This reduces I/O load on the main set of disks.
42. What is use of Rollback Segments In Database ?
They allow the database to maintain read consistency between multiple transactions.
43. What is a Rollback segment entry ?
It is the set of before image data blocks that contain rows that are modified by a transaction.Each Rollback Segment entry must be completed within one rollback segment.
A single rollback segment can have multiple rollback segment entries.
44. What is hit ratio ?
It is a measure of well the data cache buffer is handling requests for data.
Hit Ratio = (Logical Reads - Physical Reads - Hits Misses)/ Logical Reads.
45. When will be a segment released ?
When Segment is dropped.When Shrink (RBS only)When truncated (TRUNCATE used with drop storage option)
46. What are disadvanteges of having raw devices ?
We should depend on export/import utility for backup/recovery (fully reliable)
The tar command cannot be used for physical file backup, instead we can use dd command which is less flexible and has limited recoveries.
47. List the factors that can affect the accuracy of the estimations ?
The space used transaction entries and deleted records does not become free immediately after completion due to delayed cleanout.
Trailling nulls and length bytes are not stored.
Inserts of, updates to and deletes of rows as well as columns larger than a single datablock, can cause fragmentation an chained row pieces.
DATABASE SECURITY & ADMINISTRATION
48. What is user Account in Oracle database ?
An user account is not a physical structure in Database but it is having important relationship to the objects in the database and will be having certain privileges.
49. How will you enforce security using stored procedures ?
Don't grant user access directly to tables within the application.
Instead grant the ability to access the procedures that access the tables.
When procedure executed it will execute the privilege of procedures owner. Users cannot access tables except via the procedure.
50. What are the dictionary tables used to monitor a database spaces ?
DBA_FREE_SPACEDBA_SEGMENTSDBA_DATA_FILES.
51. What are the responsibilities of a Database Administrator ?
Installing and upgrading the Oracle Server and application tools.Allocating system storage and planning future storage requirements for the database system.Managing primary database structures (tablespaces)Managing primary objects (table,views,indexes)Enrolling users and maintaining system security.Ensuring compliance with Oralce license agreementControlling and monitoring user access to the database.Monitoring and optimising the performance of the database.Planning for backup and recovery of database information.Maintain archived data on tapeBacking up and restoring the database.Contacting Oracle Corporation for technical support.
52. What are the roles and user accounts created automatically with the database ?
DBA - role Contains all database system privileges.
SYS user account - The DBA role will be assigned to this account. All of the basetables and views for the database's dictionary are store in this schema and are manipulated only by ORACLE.
SYSTEM user account - It has all the system privileges for the database and additional tables and views that display administrative information and internal tables and views used by oracle tools are created using this username.
54. What are the database administrators utilities avaliable ?
SQL * DBA - This allows DBA to monitor and control an ORACLE database.
SQL * Loader - It loads data from standard operating system files (Flat files) into ORACLE database tables.
Export (EXP) and Import (imp) utilities allow you to move existing data in ORACLE format to and from ORACLE database.
55. What are the minimum parameters should exist in the parameter file (init.ora) ?
DB NAME - Must set to a text string of no more than 8 characters and it will be stored inside the datafiles, redo log files and control files and control file while database creation.
DB_DOMAIN - It is string that specifies the network domain where the database is created. The global database name is identified by setting these parameters (DB_NAME & DB_DOMAIN)
CONTORL FILES - List of control filenames of the database. If name is not mentioned then default name will be used.
DB_BLOCK_BUFFERS - To determine the no of buffers in the buffer cache in SGA.
PROCESSES - To determine number of operating system processes that can be connected to ORACLE concurrently. The value should be 5 (background process) and additional 1 for each user.
ROLLBACK_SEGMENTS - List of rollback segments an ORACLE instance acquires at database startup.
Also optionally LICENSE_MAX_SESSIONS,LICENSE_SESSION_WARNING and LICENSE_MAX_USERS.
56. What is a trace file and how is it created ?
Each server and background process can write an associated trace file. When an internal error is detected by a process or user process, it dumps information about the error to its trace. This can be used for tuning the database.
57. What are roles ? How can we implement roles ?
Roles are the easiest way to grant and manage common privileges needed by different groups of database users.
Creating roles and assigning provies to roles.
Assign each role to group of users. This will simplify the job of assigning privileges to individual users.
58. What are the steps to switch a database's archiving mode between NO ARCHIVELOG and ARCHIVELOG mode ?
1. Shutdown the database instance.2. Backup the databse3. Perform any operating system specific steps (optional)4. Start up a new instance and mount but do not open the databse.5. Switch the databse's archiving mode.
59. How can you enable automatic archiving ?
Shut the databaseBackup the databaseModify/Include LOG_ARCHIVE_START_TRUE in init.ora file.Start up the databse.
60. How can we specify the Archived log file name format and destination ?
By setting the following values in init.ora file.
LOG_ARCHIVE_FORMAT = arch %S/s/T/tarc (%S - Log sequence number and is zero left paded, %s - Log sequence number not padded. %T - Thread number lef-zero-paded and %t - Thread number not padded). The file name created is arch 0001 are if %S is used.LOG_ARCHIVE_DEST = path.
61. What is the use of ANALYZE command ?
To perform one of these function on an index,table, or cluster:
- to collect statisties about object used by the optimizer and store them in the data dictionary.- to delete statistics about the object used by object from the data dictionary.- to validate the structure of the object.- to identify migrated and chained rows of the table or cluster.
MANAGING DISTRIBUTED DATABASES.
62. How can we reduce the network traffic ?- Replictaion of data in distributed environment.- Using snapshots to replicate data.- Using remote procedure calls.
63. What is snapshots ?
Snapshot is an object used to dynamically replicate data between distribute database at specified time intervals. In ver 7.0 they are read only.
64. What are the various type of snapshots ?
Simple and Complex.
65. Differentiate simple and complex, snapshots ?
- A simple snapshot is based on a query that does not contains GROUP BY clauses, CONNECT BY clauses, JOINs, sub-query or snashot of operations.- A complex snapshots contain atleast any one of the above.
66. What dynamic data replication ?
Updating or Inserting records in remote database through database triggers. It may fail if remote database is having any problem.
67. How can you Enforce Refrencial Integrity in snapshots ?
Time the references to occur when master tables are not in use.Peform the reference the manually immdiately locking the master tables. We can join tables in snopshots by creating a complex snapshots that will based on the master tables.
68. What are the options available to refresh snapshots ?
COMPLETE - Tables are completly regenerated using the snapshot's query and the master tables every time the snapshot referenced.FAST - If simple snapshot used then a snapshot log can be used to send the changes to the snapshot tables.FORCE - Default value. If possible it performs a FAST refresh; Otherwise it will perform a complete refresh.
69. what is snapshot log ?
It is a table that maintains a record of modifications to the master table in a snapshot. It is stored in the same database as master table and is only available for simple snapshots. It should be created before creating snapshots.
70. When will the data in the snapshot log be used ?
We must be able to create a after row trigger on table (i.e., it should be not be already available )
After giving table privileges.
We cannot specify snapshot log name because oracle uses the name of the master table in the name of the database objects that support its snapshot log.
The master table name should be less than or equal to 23 characters.
(The table name created will be MLOGS_tablename, and trigger name will be TLOGS name).
72. What are the benefits of distributed options in databases ?
Database on other servers can be updated and those transactions can be grouped together with others in a logical unit.Database uses a two phase commit.
MANAGING BACKUP & RECOVERY
73. What are the different methods of backing up oracle database ?
- Logical Backups- Cold Backups- Hot Backups (Archive log)
74. What is a logical backup ?
Logical backup involves reading a set of databse records and writing them into a file. Export utility is used for taking backup and Import utility is used to recover from backup.
75. What is cold backup ? What are the elements of it ?
Cold backup is taking backup of all physical files after normal shutdown of database. We need to take.- All Data files.- All Control files.- All on-line redo log files.- The init.ora file (Optional)
76. What are the different kind of export backups ?
Full back - Complete databaseIncremental - Only affected tables from last incremental date/full backup date.Cumulative backup - Only affected table from the last cumulative date/full backup date.
77. What is hot backup and how it can be taken ?
Taking backup of archive log files when database is open. For this the ARCHIVELOG mode should be enabled. The following files need to be backed up. All data files. All Archive log, redo log files. All control files.
78. What is the use of FILE option in EXP command ?
To give the export file name.
79. What is the use of COMPRESS option in EXP command ?
Flag to indicate whether export should compress fragmented segments into single extents.
80. What is the use of GRANT option in EXP command ?
A flag to indicate whether grants on databse objects will be exported or not. Value is 'Y' or 'N'.
81. What is the use of INDEXES option in EXP command ?
A flag to indicate whether indexes on tables will be exported.
82. What is the use of ROWS option in EXP command ?
Flag to indicate whether table rows should be exported. If 'N' only DDL statements for the databse objects will be created.
83. What is the use of CONSTRAINTS option in EXP command ?
A flag to indicate whether constraints on table need to be exported.
84. What is the use of FULL option in EXP command ?
A flag to indicate whether full databse export should be performed.
85. What is the use of OWNER option in EXP command ?List of table accounts should be exported.
86. What is the use of TABLES option in EXP command ?
List of tables should be exported.
87. What is the use of RECORD LENGTH option in EXP command ?
Record length in bytes.
88. What is the use of INCTYPE option in EXP command ?
Type export should be performed COMPLETE,CUMULATIVE,INCREMENTAL.
89. What is the use of RECORD option in EXP command ?
For Incremental exports, the flag indirects whether a record will be stores data dictionary tables recording the export.
90. What is the use of PARFILE option in EXP command ?
Name of the parameter file to be passed for export.
91. What is the use of PARFILE option in EXP command ?
Name of the parameter file to be passed for export.
92. What is the use of ANALYSE ( Ver 7) option in EXP command ?
A flag to indicate whether statistical information about the exported objects should be written to export dump file.
93. What is the use of CONSISTENT (Ver 7) option in EXP command ?
A flag to indicate whether a read consistent version of all the exported objects should be maintained.
94. What is use of LOG (Ver 7) option in EXP command ?
The name of the file which log of the export will be written.
95.What is the use of FILE option in IMP command ?
The name of the file from which import should be performed.
96. What is the use of SHOW option in IMP command ?
A flag to indicate whether file content should be displayed or not.
97. What is the use of IGNORE option in IMP command ?
A flag to indicate whether the import should ignore errors encounter when issuing CREATE commands.
98. What is the use of GRANT option in IMP command ?
A flag to indicate whether grants on database objects will be imported.
99. What is the use of INDEXES option in IMP command ?
A flag to indicate whether import should import index on tables or not.
100. What is the use of ROWS option in IMP command ?
A flag to indicate whether rows should be imported. If this is set to 'N' then only DDL for database objects will be exectued.
SQL PLUS STATEMENTS
1. What are the types of SQL Statement ?
Data Definition Language : CREATE,ALTER,DROP,TRUNCATE,REVOKE,NO AUDIT & COMMIT.Data Manipulation Language : INSERT,UPDATE,DELETE,LOCK TABLE,EXPLAIN PLAN & SELECT.Transactional Control : COMMIT & ROLLBACKSession Control : ALTERSESSION & SET ROLESystem Control : ALTER SYSTEM.
2. What is a transaction ?
Transaction is logical unit between two commits and commit and rollback.
3. What is difference between TRUNCATE & DELETE ?
TRUNCATE commits after deleting entire table i.e., can not be rolled back. Database triggers do not fire on TRUNCATE
DELETE allows the filtered deletion. Deleted records can be rolled back or committed.Database triggers fire on DELETE.
4. What is a join ? Explain the different types of joins ?
Join is a query which retrieves related columns or rows from multiple tables.
Self Join - Joining the table with itself.Equi Join - Joining two tables by equating two common columns.Non-Equi Join - Joining two tables by equating two common columns.Outer Join - Joining two tables in such a way that query can also retrive rows that do not have corresponding join value in the other table.
5. What is the Subquery ?
Subquery is a query whose return values are used in filtering conditions of the main query.
6. What is correlated sub-query ?
Correlated sub_query is a sub_query which has reference to the main query.
7. Explain Connect by Prior ?
Retrives rows in hierarchical order.e.g. select empno, ename from emp where.
8. Difference between SUBSTR and INSTR ?
INSTR (String1,String2(n,(m)),INSTR returns the position of the mth occurrence of the string 2 instring1. The search begins from nth position of string1.
SUBSTR (String1 n,m)SUBSTR returns a character string of size m in string1, starting from nth postion of string1.
9. Explain UNION,MINUS,UNION ALL, INTERSECT ?
INTERSECT returns all distinct rows selected by both queries.MINUS - returns all distinct rows selected by the first query but not by the second.UNION - returns all distinct rows selected by either queryUNION ALL - returns all rows selected by either query,including all duplicates.
10. What is ROWID ?
ROWID is a pseudo column attached to each row of a table. It is 18 character long, blockno, rownumber are the components of ROWID.
11. What is the fastest way of accessing a row in a table ?
Using ROWID.
CONSTRAINTS
12. What is an Integrity Constraint ?
Integrity constraint is a rule that restricts values to a column in a table.
13. What is Referential Integrity ?
Maintaining data integrity through a set of rules that restrict the values of one or more columns of the tables based on the values of primary key or unique key of the referenced table.
14. What are the usage of SAVEPOINTS ?
SAVEPOINTS are used to subdivide a transaction into smaller parts. It enables rolling back part of a transaction. Maximum of five save points are allowed.
15. What is ON DELETE CASCADE ?
When ON DELETE CASCADE is specified ORACLE maintains referential integrity by automatically removing dependent foreign key values if a referenced primary or unique key value is removed.
16. What are the data types allowed in a table ?
CHAR,VARCHAR2,NUMBER,DATE,RAW,LONG and LONG RAW.
17. What is difference between CHAR and VARCHAR2 ? What is the maximum SIZE allowed for each type ?
CHAR pads blank spaces to the maximum length. VARCHAR2 does not pad blank spaces. For CHAR it is 255 and 2000 for VARCHAR2.
18. How many LONG columns are allowed in a table ? Is it possible to use LONG columns in WHERE clause or ORDER BY ?
Only one LONG columns is allowed. It is not possible to use LONG column in WHERE or ORDER BY clause.
19. What are the pre requisites ?I. to modify datatype of a column ?ii. to add a column with NOT NULL constraint ?
To Modify the datatype of a column the column must be empty.to add a column with NOT NULL constrain, the table must be empty.
20. Where the integrity constrints are stored in Data Dictionary ?
The integrity constraints are stored in USER_CONSTRAINTS.
21. How will you a activate/deactivate integrity constraints ?
The integrity constraints can be enabled or disabled by ALTER TABLE ENABLE constraint/DISABLE constraint.
22. If an unique key constraint on DATE column is created, will it validate the rows that are inserted with SYSDATE ?
It won't, Because SYSDATE format contains time attached with it.
23. What is a database link ?
Database Link is a named path through which a remote database can be accessed.
24. How to access the current value and next value from a sequence ? Is it possible to access the current value in a session before accessing next value ?
Sequence name CURRVAL, Sequence name NEXTVAL.
It is not possible. Only if you access next value in the session, current value can be accessed.
25. What is CYCLE/NO CYCLE in a Sequence ?
CYCLE specifies that the sequence continues to generate values after reaching either maximum or minimum value. After pan ascending sequence reaches its maximum value, it generates its minimum value. After a descending sequence reaches its minimum, it generates its maximum.
NO CYCLE specifies that the sequence cannot generate more values after reaching its maximum or minimum value.
26. What are the advantages of VIEW ?
To protect some of the columns of a table from other users.To hide complexity of a query.To hide complexity of calculations.
27. Can a view be updated/inserted/deleted? If Yes under what conditions ?
A View can be updated/deleted/inserted if it has only one base table if the view is based on columns from one or more tables then insert, update and delete is not possible.
28.If a View on a single base table is manipulated will the changes be reflected on the base table ?
If changes are made to the tables which are base tables of a view will the changes be reference on the view.
FORMS 3.0 BASIC
1.What is an SQL *FORMS ?
SQL *forms is 4GL tool for developing and executing; Oracle based interactive application.
2. What is the maximum size of a form ?
255 character width and 255 characters Length.
3. Name the two files that are created when you generate the form give the filex extension ?
INP (Source File)FRM (Executable File)
4. How do you control the constraints in forms ?
Select the use constraint property is ON Block definition screen.
BLOCK
5. Commited block sometimes refer to a BASE TABLE ? True or False.
False.
6. Can we create two blocks with the same name in form 3.0 ?
No.
7. While specifying master/detail relationship between two blocks specifying the join condition is a must ? True or False.
True.
8. What is a Trigger ?
A piece of logic that is executed at or triggered by a SQL *forms event.
9. What are the types of TRIGGERS ?
1. Navigational Triggers.2. Transaction Triggers.
10. What are the different types of key triggers ?
Function KeyKey-functionKey-othersKey-startup
11. What is the difference between a Function Key Trigger and Key Function Trigger ?
Function key triggers are associated with individual SQL*FORMS function keysYou can attach Key function triggers to 10 keys or key sequences that normally do not perform any SQL * FORMS operations. These keys refered as key F0 through key F9.
12. What does an on-clear-block Trigger fire?
It fires just before SQL * forms the current block.
13. How do you trap the error in forms 3.0 ?
using On-Message or On-Error triggers.
14. State the order in which these triggers are executed ?
POST-FIELD,ON-VALIDATE-FIELD,POST-CHANGE and KEY-NEXTFLD.KEY-NEXTFLD,POST-CHANGE, ON-VALIDATE-FIELD, POST-FIELD.
15. What is the usuage of an ON-INSERT,ON-DELETE and ON-UPDATE TRIGGERS ?
These triggers are executes when inserting,deleting and updating operations are performed and can be used to change the default function of insert,delete or update respectively.
For Eg, instead of inserting a row in a table an existing row can be updated in the same table.
16. When will ON-VALIDATE-FIELD trigger executed ?
It fires when a value in a field has been changed and the field status is changed or new and the key has been pressed. If the field status is valid then any further change to the value in the field will not fire the on-validate-field trigger.
17. A query fetched 10 records How many times does a PRE-QUERY Trigger and POST-QUERY Trigger will get executed ?
PRE-QUERY fires once.POST-QUERY fires 10 times.
18. What is the difference between ON-VALIDATE-FIELD trigger and a POST-CHANGE trigger ?
When you changes the Existing value to null, the On-validate field trigger will fire post change trigger will not fire. At the time of execute-query post-chage trigger will fire, on-validate field trigger will not fire.
19. What is the difference between an ON-VALIDATE-FIELD trigger and a trigger ?
On-validate-field trigger fires, when the field Validation status New or changed.Post-field-trigger whenever the control leaving form the field, it will fire.
20. What is the difference between a POST-FIELD trigger and a POST-CHANGE trigger ?
Post-field trigger fires whenever the control leaving from the filed.Post-change trigger fires at the time of execute-query procedure invoked or filed validation status changed.
21. When is PRE-QUERY trigger executed ?
When Execute-query or count-query Package procedures are invoked.
22. Give the sequence in which triggers fired during insert operations, when the following 3 triggers are defined at the smae block level ?a. ON-INSERT b. POST-INSERT c. PRE-INSERT
PRE-INSERT,ON-INSERT & POST-INSERT.
23. Can we use GO-BLOCK package in a pre-field trigger ?
No.
24. Is a Keystartup trigger fires as result of a operator pressing a key explicitly ?
No.
25. How can you execute the user defined triggers in forms 3.0 ?
Execute_Trigger (trigger-name)
26. When does an on-lock trigger fire ?
It will fires whenever SQL * Forms would normally attempt to lock a row.
26. What is Post-Block is a. a. Navigational Trigger.b. Key triggerc. Transaction Trigger.
Navigational Trigger.
27. What is the difference between keystartup and pre-form ?
Key-startup trigger fires after successful navigation into a form.
Pre-form trigger fires before enter into the form.
28. What is the difference between keystartup and pre-form ?
Key-startup triigger fires after successful navigation into a form.Pre-form trigger fires before enter into the form.
PACKAGE PROCEDURE & FUNCTION
29. What is a Package Procedure ?
A Package proecdure is built in PL/SQL procedure.
30. What are the different types of Package Procedure ?
1. Restricted package procedure.2. Unrestricted package proecdure.
31. What is the difference between restricted and unrestricted package procedure ?Restricted package procedure that affects the basic basic functions of SQL * Forms. It cannot used in all triggers execpt key triggers.
Unrestricted package procedure that does not interfere with the basic functions of SQL * Forms it can be used in any triggers.
32. Classify the restricted and unrestricted procedure from the following.a. Callb. User-Exitc. Call-Queryd. Upe. Execute-Queryf. Messageg. Exit-Fromh. Posti. Break
a. Call - unrestrictedb. User Exit - Unrestrictedc. Call_query - Unrestrictedd. Up - Restrictede. Execute Query - Restrictedf. Message - Restrictedg. Exit_form - Restrictedh. Post - Restrictedi. Break - Unrestricted.
33. Can we use a restricted package procedure in ON-VALIDATE-FIELD Trigger ?
No.
34. What SYNCHRONIZE procedure does ?
It synchoronizes the terminal screen with the internal state of the form.
35. What are the unrestricted procedures used to change the popup screen position during run time ?
Anchor-viewResize -ViewMove-View.
36. What Enter package procedure does ?
Enter Validate-data in the current validation unit.
37. What ERASE package procedure does ?
Erase removes an indicated global variable.
38. What is the difference between NAME_IN and COPY ?
Copy is package procedure and writes values into a field.Name in is a package function and returns the contents of the variable to which you apply.
38. Identify package function from the following ?1. Error-Code2. Break3. Call4. Error-text5. Form-failure6. Form-fatal7. Execute-query8. Anchor_View9. Message_code
1. Error_Code2. Error_Text3. Form_Failure4. Form_Fatal5. Message_Code
40. How does the command POST differs from COMMIT ?
Post writes data in the form to the database but does not perform database commitCommit permenently writes data in the form to the database.
41. What the PAUSE package procedure does ?
Pause suspends processing until the operator presses a function key
42. What package procedure is used for calling another form ?
Call (E.g. Call(formname)
43. What package procedure used for invoke sql *plus from sql *forms ?
Host (E.g. Host (sqlplus))
44. Error_Code is a package proecdure ?a. True b. false
False.
45. EXIT_FORM is a restricted package procedure ?a. True b. False
True.
46. When the form is running in DEBUG mode, If you want to examine the values of global variables and other form variables, What package procedure command you would use in your trigger text ?
Break.
SYSTEM VARIABLES
47. List the system variables related in Block and Field?
1. System.block_status2. System.current_block3. System.current_field4. System.current_value5. System.cursor_block6. System.cursor_field7. System.field_status.
48. What is the difference between system.current_field and system.cursor_field ?
1. System.current_field gives name of the field.2. System.cursor_field gives name of the field with block name.
49. The value recorded in system.last_record variable is of typea. Numberb. Booleanc. Character.b. Boolean.
User Exits :
50. What is an User Exits ?
A user exit is a subroutine which are written in programming languages using pro*C pro *Cobol , etc., that link into the SQL * forms executable.
51. What are the type of User Exits ?
ORACLE Precompliers user exitsOCI (ORACLE Call Interface)Non-ORACEL user exits.
Page :
52. What do you mean by a page ?
Pages are collection of display information, such as constant text and graphics.
53. How many pages you can in a single form ?
Unlimited.
54. Two popup pages can appear on the screen at a time ?a. True b. False
a. True.
55.What is the significance of PAGE 0 in forms 3.0 ?
Hide the fields for internal calculation.
56. Deleting a page removes information about all the fields in that page ?a. True. b. False
a. True.
Popup Window :
57. What do you mean by a pop-up window ?
Pop-up windows are screen areas that overlay all or a portion of thedisplay screen when a form is running.
58. What are the types of Pop-up window ?
the pop-up field editorpop-up list of valuespop-up pages.
Alert :
59. What is an Alert ?
An alert is window that appears in the middle of the screen overlaying a portion of the current display.
FORMS 4.0
01. Give the Types of modules in a form?
FormMenuLibrary
02. Write the Abbreviation for the following File Extension1. FMB 2. MMB 3. PLL
FMB ----- Form Module Binary.MMB ----- Menu Module Binary.PLL ------ PL/SQL Library Module Binary.
03. What are the design facilities available in forms 4.0?
Default Block facility.Layout Editor.Menu Editor.Object Lists.Property Sheets.PL/SQL Editor.Tables Columns Browser.Built-ins Browser.
04. What is a Layout Editor?
The Layout Editor is a graphical design facility for creating and arranging items and boilerplate text and graphics objects in your application's interface.
05. BLOCK
05. What do you mean by a block in forms4.0?
Block is a single mechanism for grouping related items into a functional unit for storing,displaying and manipulating records.
06. Explain types of Block in forms4.0?
Base table Blocks.Control Blocks.1. A base table block is one that is associated with a specific database table or view.2. A control block is a block that is not associated with a database table.
ITEMS
07. List the Types of Items?
Text item.Chart item.Check box.Display item.Image item.List item.Radio Group.User Area item.
08. What is a Navigable item?
A navigable item is one that operators can navigate to with the keyboard during default navigation, or that Oracle forms can navigate to by executing a navigationalbuilt-in procedure.
09. Can you change the color of the push button in design time?
No.
10. What is a Check Box?
A Check Box is a two state control that indicates whether a certain condition or value is on or off, true or false. The display state of a check box is always either "checked" or "unchecked".
11. What are the triggers associated with a check box?
Only When-checkbox-activated Trigger associated with a Check box.
PL/SQL
Basiscs of PL/SQL
1. What is PL/SQL ?PL/SQL is a procedural language that has both interactive SQL and procedural programming language constructs such as iteration, conditional branching.
2. What is the basic structure of PL/SQL ?
PL/SQL uses block structure as its basic structure. Anonymous blocks or nested blocks can be used in PL/SQL.
3. What are the components of a PL/SQL block ?
A set of related declarations and procedural statements is called block.
4. What are the components of a PL/SQL Block ?
Declarative part, Executable part and Execption part.
Datatypes PL/SQL
5. What are the datatypes a available in PL/SQL ?
Some scalar data types such as NUMBER, VARCHAR2, DATE, CHAR, LONG, BOOLEAN.Some composite data types such as RECORD & TABLE.
6. What are % TYPE and % ROWTYPE ? What are the advantages of using these over datatypes?
% TYPE provides the data type of a variable or a database column to that variable.
% ROWTYPE provides the record type that represents a entire row of a table or view or columns selected in the cursor.
The advantages are : I. Need not know about variable's data typeii. If the database definition of a column in a table changes, the data type of a variable changes accordingly.
7. What is difference between % ROWTYPE and TYPE RECORD ?
% ROWTYPE is to be used whenever query returns a entire row of a table or view.
TYPE rec RECORD is to be used whenever query returns columns of differenttable or views and variables.
E.g. TYPE r_emp is RECORD (eno emp.empno% type,ename emp ename %type);e_rec emp% ROWTYPEcursor c1 is select empno,deptno from emp;e_rec c1 %ROWTYPE.
8. What is PL/SQL table ?
Objects of type TABLE are called "PL/SQL tables", which are modelled as (but not the same as) database tables, PL/SQL tables use a primary PL/SQL tables can have one column and a primary key.
Cursors
9. What is a cursor ? Why Cursor is required ?
Cursor is a named private SQL area from where information can be accessed. Cursors are required to process rows individually for queries returning multiple rows.
10. Explain the two type of Cursors ?
There are two types of cursors, Implict Cursor and Explicit Cursor.PL/SQL uses Implict Cursors for queries.User defined cursors are called Explicit Cursors. They can be declared and used.
11. What are the PL/SQL Statements used in cursor processing ?
DECLARE CURSOR cursor name, OPEN cursor name, FETCH cursor name INTO or Record types, CLOSE cursor name.
12. What are the cursor attributes used in PL/SQL ?
%ISOPEN - to check whether cursor is open or not% ROWCOUNT - number of rows featched/updated/deleted.% FOUND - to check whether cursor has fetched any row. True if rows are featched.% NOT FOUND - to check whether cursor has featched any row. True if no rows are featched.These attributes are proceded with SQL for Implict Cursors and with Cursor name for Explict Cursors.
13. What is a cursor for loop ?
Cursor for loop implicitly declares %ROWTYPE as loop index,opens a cursor, fetches rows of values from active set into fields in the record and closeswhen all the records have been processed.
eg. FOR emp_rec IN C1 LOOPsalary_total := salary_total +emp_rec sal;END LOOP;
14. What will happen after commit statement ?Cursor C1 isSelect empno,ename from emp;Beginopen C1; loopFetch C1 intoeno.ename;Exit WhenC1 %notfound;-----commit;end loop;end;
The cursor having query as SELECT .... FOR UPDATE gets closed after COMMIT/ROLLBACK.
The cursor having query as SELECT.... does not get closed even after COMMIT/ROLLBACK.
15. Explain the usage of WHERE CURRENT OF clause in cursors ?
WHERE CURRENT OF clause in an UPDATE,DELETE statement refers to the latest row fetched from a cursor.
Database Triggers
16. What is a database trigger ? Name some usages of database trigger ?
Database trigger is stored PL/SQL program unit associated with a specific database table. Usages are Audit data modificateions, Log events transparently, Enforce complex business rules Derive column values automatically, Implement complex security authorizations. Maintain replicate tables.
17. How many types of database triggers can be specified on a table ? What are they ?
Insert Update Delete
Before Row o.k. o.k. o.k.
After Row o.k. o.k. o.k.
Before Statement o.k. o.k. o.k.
After Statement o.k. o.k. o.k.
If FOR EACH ROW clause is specified, then the trigger for each Row affected by the statement.
If WHEN clause is specified, the trigger fires according to the retruned boolean value.
18. Is it possible to use Transaction control Statements such a ROLLBACK or COMMIT in Database Trigger ? Why ?
It is not possible. As triggers are defined for each table, if you use COMMIT of ROLLBACK in a trigger, it affects logical transaction processing.
19. What are two virtual tables available during database trigger execution ?
The table columns are referred as OLD.column_name and NEW.column_name.For triggers related to INSERT only NEW.column_name values only available.For triggers related to UPDATE only OLD.column_name NEW.column_name values only available.For triggers related to DELETE only OLD.column_name values only available.
20. What happens if a procedure that updates a column of table X is called in a database trigger of the same table ?
Mutation of table occurs.
21. Write the order of precedence for validation of a column in a table ?I. done using Database triggers.ii. done using Integarity Constraints.
I & ii.
Exception :
22. What is an Exception ? What are types of Exception ?
Exception is the error handling part of PL/SQL block. The types are Predefined and user_defined. Some of Predefined execptions are. CURSOR_ALREADY_OPENDUP_VAL_ON_INDEXNO_DATA_FOUNDTOO_MANY_ROWSINVALID_CURSORINVALID_NUMBERLOGON_DENIEDNOT_LOGGED_ONPROGRAM-ERRORSTORAGE_ERRORTIMEOUT_ON_RESOURCEVALUE_ERRORZERO_DIVIDEOTHERS.
23. What is Pragma EXECPTION_INIT ? Explain the usage ?
The PRAGMA EXECPTION_INIT tells the complier to associate an exception with an oracle error. To get an error message of a specific oracle error.
e.g. PRAGMA EXCEPTION_INIT (exception name, oracle error number)
24. What is Raise_application_error ?
Raise_application_error is a procedure of package DBMS_STANDARD which allows to issue an user_defined error messages from stored sub-program or database trigger.
25. What are the return values of functions SQLCODE and SQLERRM ?
SQLCODE returns the latest code of the error that has occured.SQLERRM returns the relevant error message of the SQLCODE.
26. Where the Pre_defined_exceptions are stored ?
In the standard package.
Procedures, Functions & Packages ;
27. What is a stored procedure ?
A stored procedure is a sequence of statements that perform specific function.
28. What is difference between a PROCEDURE & FUNCTION ?
A FUNCTION is alway returns a value using the return statement.A PROCEDURE may return one or more values through parameters or may not return at all.
29. What are advantages fo Stored Procedures /
Extensibility,Modularity, Reusability, Maintainability and one time compilation.
30. What are the modes of parameters that can be passed to a procedure ?
IN,OUT,IN-OUT parameters.
31. What are the two parts of a procedure ?
Procedure Specification and Procedure Body.
32. Give the structure of the procedure ?
PROCEDURE name (parameter list.....)islocal variable declarations
BEGINExecutable statements.Exception.exception handlers
end;
33. Give the structure of the function ?
FUNCTION name (argument list .....) Return datatype islocal variable declarationsBeginexecutable statementsExceptionexecution handlersEnd;
34. Explain how procedures and functions are called in a PL/SQL block ?
Function is called as part of an expression.sal := calculate_sal ('a822');procedure is called as a PL/SQL statementcalculate_bonus ('A822');
35. What is Overloading of procedures ?
The Same procedure name is repeated with parameters of different datatypes and parameters in different positions, varying number of parameters is called overloading of procedures.
e.g. DBMS_OUTPUT put_line
36. What is a package ? What are the advantages of packages ?
Package is a database object that groups logically related procedures.The advantages of packages are Modularity, Easier Applicaton Design, Information. Hiding,. reusability and Better Performance.
37.What are two parts of package ?
The two parts of package are PACKAGE SPECIFICATION & PACKAGE BODY.
Package Specification contains declarations that are global to the packages and local to the schema.Package Body contains actual procedures and local declaration of the procedures and cursor declarations.
38. What is difference between a Cursor declared in a procedure and Cursor declared in a package specification ?
A cursor declared in a package specification is global and can be accessed by other procedures or procedures in a package.A cursor declared in a procedure is local to the procedure that can not be accessed by other procedures.
39. How packaged procedures and functions are called from the following?a. Stored procedure or anonymous blockb. an application program such a PRC *C, PRO* COBOLc. SQL *PLUS
a. PACKAGE NAME.PROCEDURE NAME (parameters);variable := PACKAGE NAME.FUNCTION NAME (arguments);EXEC SQL EXECUTEb.BEGINPACKAGE NAME.PROCEDURE NAME (parameters)variable := PACKAGE NAME.FUNCTION NAME (arguments);END;END EXEC;c. EXECUTE PACKAGE NAME.PROCEDURE if the procedures does not have anyout/in-out parameters. A function can not be called.
40. Name the tables where characteristics of Package, procedure and functions are stored ?
User_objects, User_Source and User_error.
FORMS4.0
12. what is a display item?
Display items are similar to text items but store only fetched or assigned values. Operators cannot navigate to a display item or edit the value it contains.
13. What is a list item?
It is a list of text elements.
14. What are the display styles of list items?
Poplist, No text Item displayed in the list item.Tlist, No element in the list is highlighted.
15. What is a radio Group?
Radio groups display a fixed no of options that are mutually Exclusive .User can select one out of n number of options.
16. How many maximum number of radio buttons can you assign to a radio group?
Unlimited no of radio buttons can be assigned to a radio group
17. can you change the default value of the radio button group at run time?
No.
18.What triggers are associated with the radio group?
Only when-radio-changed trigger associated with radio group
Visual Attributes.
19. What is a visual attribute?
Visual Attributes are the font, color and pattern characteristics of objects that operators see and intract with in our application.
20. What are the types of visual attribute settings?
Custom Visual attributesDefault visual attributesNamed Visual attributes.
Window
21. What is a window?
A window, byitself , can be thought of as an empty frame. The frame provides a way to intract with the window, including the ability to scroll, move, and resize the window. The content of the window ie. what is displayed inside the frame is determined by the canvas View or canvas-views displayed in the window at run-time.
22. What are the differrent types of windows?
Root window, secondary window.
23. Can a root window be made modal?
No.
24. List the buil-in routine for controlling window during run-time?
Find_window,get_window_property,hide_window,move_window,resize_window,set_window_property,show_View
25. List the windows event triggers available in Forms 4.0?
When-window-activated, when-window-closed, when-window-deactivated,when-window-resized
26. What built-in is used for changing the properties of the window dynamically?
Set_window_property
Canvas-View
27. What is a canvas-view?
A canvas-view is the background object on which you layout the interface items (text-items, check boxes, radio groups, and so on.) and boilerplate objects that operators see and interact with as they run your form. At run-time, operators can see only those items that have been assiged to a specific canvas. Each canvas, in term, must be displayed in a specfic window.
28. Give the equivalent term in forms 4.0 for the following.Page, Page 0?
Page - Canvas-ViewPage 0 - Canvas-view null.
29. What are the types of canvas-views?
Content View, Stacked View.
30. What is the content view and stacked view?
A content view is the "Base" view that occupies the entire content pane of the window in which it is displayed.A stacked view differs from a content canvas view in that it is not the base view for the window to which it is assigned
31. List the built-in routines for the controlling canvas views during run-time?
Find_canvasGet-Canvas_propertyGet_view_propertyHide_ViewReplace_content_viewScroll_viewSet_canvas_propertySet_view_propertyShow_view
Alert
32. What is an Alert?
An alert is a modal window that displays a message notifies the operator of some application condition
33. What are the display styles of an alert?
Stop, Caution, note
34. Can you attach an alert to a field?
No
35. What built-in is used for showing the alert during run-time?
Show_alert.
36. Can you change the alert messages at run-time?If yes, give the name of th built-in to chage the alert messages at run-time.
Yes. Set_alert_property.
37. What is the built-in function used for finding the alert?
Find_alert
Editors
38. List the editors availables in forms 4.0?
Default editorUser_defined editorssystem editors.
39. What buil-in routines are used to display editor dynamicaly?
Edit_text itemshow_editor
LOV
40. What is an Lov?
A list of values is a single or multi column selection list displayed ina pop-up window
41. Can you attach an lov to a field at design time?
Yes.
42. Can you attach an lov to a field at run-time? if yes, give the build-in name.
Yes. Set_item_proprety
43. What is the built-in used for showing lov at runtime?
Show_lov
44. What is the built-in used to get and set lov properties during run-time?
Get_lov_propertySet_lov_property
Record Group
45. What is a record Group?
A record group is an internal oracle forms data structure that has a simillar column/row frame work to a database table
46. What are the different type of a record group?
Query record groupStatic record groupNon query record group
47. Give built-in routine related to a record groups?
Create_group (Function)Create_group_from_query(Function)Delete_group(Procedure)Add_group_column(Function)Add_group_row(Procedure)Delete_group_row(Procedure)Populate_group(Function)Populate_group_with_query(Function)Set_group_Char_cell(procedure)
48. What is the built_in routine used to count the no of rows in a group?
Get_group _row_count
System Variables
49. List system variables available in forms 4.0, and not available in forms 3.0?
System.cordination_operationSystem Date_thresholdSystem.effective_DateSystem.event_windowSystem.suppress_working
50. System.effective_date system variable is read only True/False
False
51. What is a library in Forms 4.0?
A library is a collection of Pl/SQL program units, including user named procedures, functions & packages
52. Is it possible to attach same library to more than one form?
Yes
53. Explain the following file extention related to library?.pll,.lib,.pld
The library pll files is a portable design file comparable to an fmb form fileThe library lib file is a plat form specific, generated library file comparable to a fmx form fileThe pld file is Txt format file and can be used for source controlling your library files
Parameter
54. How do you pass the parameters from one form to another form?
To pass one or more parameters to a called form, the calling form must perform the following steps in a trigger or user named routine excute the create_parameter_list built_in function to programatically.Create a parameter list to execute the add parameter built_in procedure to add one or more parameters list.Execute the call_form, New_form or run_product built_in procedure and include the name or id of the parameter list to be passed to the called form.
54. What are the built-in routines is available in forms 4.0 to create and manipulate a parameter list?
Add_parameterCreate_Parameter_listDelete_parameterDestroy_parameter_listGet_parameter_attrGet_parameter_listset_parameter_attr
55. What are the two ways to incorporate images into a oracle forms application?
Boilerplate ImagesImage_items
56. How image_items can be populate to field in forms 4.0?
A fetch from a long raw database column PL/Sql assignment to executing the read_image_file built_in procedure to get an image from the file system.
57. What are the triggers associated with the image item?
When-Image-activated(Fires when the operator double clicks on an image Items)When-image-pressed(fires when the operator selects or deselects the image item)
58. List some built-in routines used to manipulate images in image_item?
Image_addImage_andImage_subtractImage_xorImage_zoom
59. What are the built_in used to trapping errors in forms 4?
Error_type return characterError_code return numberError_text return charDbms_error_code return no.Dbms_error_text return char
60. What is a predefined exception available in forms 4.0?
Raise form_trigger_failure
61. What are the menu items that oracle forms 4.0 supports?
Plain, Check,Radio, Separator, Magic
FORMS4.5
object groups
01. what ia an object groups?
An object group is a container for a group of objects, you define an object group when you want to package related objects. so that you copy or reference them in another modules.
02. what are the different objects that you cannot copy or reference in object groups?
objects of differnt modulesanother object groupsindividual block dependent itemsprogram units.
canvas views
03. what are different types of canvas views?
content canvas viewsstacked canvas viewshorizontal toolbarvertical toolbar.
04. explain about content canvas views?
Most Canvas views are content canvas views a content canvas view is the "base" view that occupies the entire content pane of the window in which it is displayed.
05. Explain about stacked canvas views?
Stacked canvas view is displayed in a window on top of, or "stacked" on the content canvas view assigned to that same window. Stacked canvas views obscure some part of the underlying content canvas view, and or often shown and hidden programmatically.
06. Explain about horizontal, Vertical tool bar canvas views?
Tool bar canvas views are used to create tool bars for individual windows Horizontal tool bars are display at the top of a window, just under its menu bar.Vertical Tool bars are displayed along the left side of a window
07. Name of the functions used to get/set canvas properties?
Get_view_property, Set_view_property
Windows
07. What is relation between the window and canvas views?
Canvas views are the back ground objects on which you place the interface items (Text items), check boxes, radio groups etc.,) and boilerplateobjects (boxes, lines, images etc.,) that operators interact with us they run your form . Each canvas views displayed in a window.
08. What are the different modals of windows?
Modalless windowsModal windows
09. What are modalless windows?
More than one modelless window can be displayed at the same time, and operators can navigate among them if your application allows them to do so . On most GUI platforms, modelless windows can also be layered to appear either in front of or behind other windows.
10. What are modal windows?
Modal windows are usually used as dialogs, and have restricted functionality compared to modelless windows. On some platforms for example operators cannot resize, scroll or iconify a modal window.
11. How do you display console on a window ?
The console includes the status line and message line, and is displayed at the bottom of the window to which it is assigned.To specify that the console should be displayed, set the console window form property to the name of any window in the form. To include the console, set console window to Null.
12. What is the remove on exit property?
For a modelless window, it determines whether oracle forms hides the window automatically when the operators navigates to an item in the another window.
13. How many windows in a form can have console?
Only one window in a form can display the console, and you cannot chage the console assignment at runtime.
14. Can you have more than one content canvas view attached with a window?
Yes.Each window you create must have atleast one content canvas view assigned to it. You can also create a window that has manipulate contant canvas view. At run time only one of the content canvas views assign to a window is displayed at a time.
15. What are the different window events activated at runtimes?
When_window_activatedWhen_window_closedWhen_window_deactivatedWhen_window_resizedWithin this triggers, you can examine the built in system variable system.event_window to determine the name of the window for which the trigger fired.
Modules
27. What are different types of modules available in oracle form?
Form module - a collection of objects and code routinesMenu modules - a collection of menus and menu item commands that together make up an application menulibrary module - a collectio of user named procedures, functions and packages that can be called from other modules in the application
18. What are the default extensions of the files careated by forms modules?
.fmb - form module binary.fmx - form module executable
19. What are the default extentions of the files created by menu module?
.mmb, .mmx
20 What are the default extension of the files created by library module?
The default file extensions indicate the library module type and storage format.pll - pl/sql library module binary
Master Detail
21. What is a master detail relationship?
A master detail relationship is an association between two base table blocks- a master block and a detail block. The relationship between the blocks reflects a primary key to foreign key relationship between the tables on which the blocks are based.
22. What is coordination Event?
Any event that makes a different record in the master block the current record is a coordination causing event.
23. What are the two phases of block coordination?
There are two phases of block coordination: the clear phase and the population phase. During, the clear phase, Oracle Forms navigates internally to the detail block and flushes the obsolete detail records. During the population phase, Oracle Forms issues a SELECT statement to repopulate the detail block with detail records associated witjh the new master record. These operations are accomplished through the execution of triggers.
24. What are Most Common types of Complex master-detail relationships?
There are three most common types of complex master-detail relationships:master with dependent detailsmaster with independent detailsdetail with two masters
25. What are the different types of Delete details we can establish in Master-Details?CascadeIsolateNon-isolote
26. What are the different defaust triggers created when Master Deletes Property is set to Non-isolated?Master Delets Property Resulting Triggers----------------------------------------------------Non-Isolated(the default) On-Check-Delete-MasterOn-Clear-DetailsOn-Populate-Details
26. Whar are the different default triggers created when Master Deletes Property is set to Cascade?Ans: Master Deletes Property Resulting Triggers---------------------------------------------------Cascading On-Clear-DetailsOn-Populate-DetailsPre-delete
28. What are the different default triggers created when Master Deletes Property is set to isolated?
Master Deletes Property Resulting Triggers---------------------------------------------------Isolated On-Clear-DetailsOn-Populate-Details
29. What are the Coordination Properties in a Master-Detail relationship?The coordination properties areDeferredAuto-QueryThese Properties determine when the population phase of blockcoordination should occur.
30. What are the different types of Coordinations of the Master with the Detail block?
42. What is the User-Named Editor?
A user named editor has the same text editing functionality as the default editor, but, becaue it is a named object, you can specify editor attributes such as windows display size, position, and title.
43. What are the Built-ins to display the user-named editor?
A user named editor can be displayed programmatically with the built in procedure SHOW-EDITOR, EDIT_TETITEM independent of any particular text item.
44. What is the difference between SHOW_EDITOR and EDIT_TEXTITEM?
Show editor is the generic built_in which accepts any editor name and takes some input string and returns modified output string. Whereas the edit_textitem built_in needs the input focus to be in the text item before the built_in is excuted.
45. What is an LOV?An LOV is a scrollable popup window that provides the operator with either a single or multi column selection list.
46. What is the basic data structure that is required for creating an LOV? Record Group.
47. What is the "LOV of Validation" Property of an item? What is the use of it?When LOV for Validation is set to True, Oracle Forms compares the current value of the text item to the values in the first column displayed in the LOV.Whenever the validation event occurs.If the value in the text item matches one of the values in the first column of the LOV, validation succeeds, the LOV is not displayed, and processing continues normally.If the value in the text item does not match one of the values in the first column of the LOV, Oracle Forms displays the LOV and uses the text item value as the search criteria to automatically reduce the list.
48. What are the built_ins used the display the LOV?
Show_lovList_values
49. What are the built-ins that are used to Attach an LOV programmatically to an item?
set_item_propertyget_item_property(by setting the LOV_NAME property)
50. What are the built-ins that are used for setting the LOV properties at runtime?
get_lov_propertyset_lov_property
51. What is a record group?
A record group is an internal Oracle Forms that structure that hs a column/row framework similar to a database table. However, unlike database tables, record groups are separate objects that belong to the form module which they are defined.
52. How many number of columns a record group can have?
A record group can have an unlimited number of columns of type CHAR, LONG, NUMBER, or DATE provided that the total number of column does not exceed 64K.
53. What is the Maximum allowed length of Record group Column?
Record group column names cannot exceed 30 characters.
54. What are the different types of Record Groups?
Query Record GroupsNonQuery Record GroupsState Record Groups
55. What is a Query Record Group?
A query record group is a record group that has an associated SELECT statement. The columns in a query record group derive their default names, data types, had lengths from the database columns referenced in the SELECT statement. The records in query record group are the rows retrieved by the query associated with that record group.
56. What is a Non Query Record Group?
A non-query record group is a group that does not have an associated query, but whose structure and values can be modified programmatically at runtime.
57. What is a Static Record Group?
A static record group is not associated with a query, rather, you define its structure and row values at design time, and they remain fixed at runtime.
58. What are the built-ins used for Creating and deleting groups?
CREATE-GROUP (function)CREATE_GROUP_FROM_QUERY(function)DELETE_GROUP(procedure)
59.What are the built -ins used for Modifying a group's structure?
ADD-GROUP_COLUMN (function)ADD_GROUP_ROW (procedure)DELETE_GROUP_ROW(procedure)
60. POPULATE_GROUP(function)POPULATE_GROUP_WITH_QUERY(function)SET_GROUP_CHAR_CELL(procedure)SET_GROUP_DATE_CELL(procedure)SET_GROUP_NUMBER_CELL(procedure)
61. What are the built-ins used for Getting cell values?
GET_GROUP_CHAR_CELL (function)GET_GROUP_DATE_CELL(function)GET_GROUP_NUMBET_CELL(function)
62. What are built-ins used for Processing rows?
GET_GROUP_ROW_COUNT(function)GET_GROUP_SELECTION_COUNT(function)GET_GROUP_SELECTION(function)RESET_GROUP_SELECTION(procedure)SET_GROUP_SELECTION(procedure)UNSET_GROUP_SELECTION(procedure)
63. What are the built-ins used for finding Object ID function?
FIND_GROUP(function)FIND_COLUMN(function)
64. Use the ADD_GROUP_COLUMN function to add a column to a record group that was created at design time.I) TRUE II)FALSE
II) FALSE
65. Use the ADD_GROUP_ROW procedure to add a row to a static record group
I) TRUE II)FALSEI) FALSE
61. What are the built-in used for getting cell values?
Get_group_char_cell(function)Get_group_date_cell(function)Get_group_number_cell(function)
62. What are the built-ins used for processing rows?
Get_group_row_count(function)Get_group_selection_count(function)Get_group_selection(function)Reset_group_selection(procedure)Set_group_selection(procedure)Unset_group_selection(procedure)
63. What are the built-ins used for finding object ID functions?
Find_group(function)Find_column(function)
64. Use the add_group_column function to add a column to record group that was created at a design time?
False.
65. Use the Add_group_row procedure to add a row to a static record group 1. true or false?
False.
parameters
66. What are parameters?
Parameters provide a simple mechanism for defining and setting the valuesof inputs that are required by a form at startup. Form parameters are variables of type char,number,date that you define at design time.
67. What are the Built-ins used for sending Parameters to forms?
You can pass parameter values to a form when an application executes the call_form, New_form, Open_form or Run_product.
68. What is the maximum no of chars the parameter can store?
The maximum no of chars the parameter can store is only valid for char parameters, which can be upto 64K. No parameters default to 23Bytes and Date parameter default to 7Bytes.
69. How do you call other Oracle Products from Oracle Forms?
Run_product is a built-in, Used to invoke one of the supported oracle tools products and specifies the name of the document or module to be run. If the called product is unavailable at the time of the call, Oracle Forms returns a message to the opertor.
70. How do you reference a Parameter?
In Pl/Sql, You can reference and set the values of form parameters using bind variables syntax. Ex. PARAMETER name = '' or :block.item = PARAMETER Parameter name
71. How do you reference a parameter indirectly?
To indirectly reference a parameter use the NAME IN, COPY 'built-ins to indirectly set and reference the parameters value' Example name_in ('capital parameter my param'), Copy ('SURESH','Parameter my_param')
72. What are the different Parameter types?
Text ParametersData Parameters
73. When do you use data parameter type?
When the value of a data parameter being passed to a called product is always the name of the record group defined in the current form. Data parameters are used to pass data to produts invoked with the run_product built-in subprogram.
74. Can you pass data parametrs to forms?
No.
IMAGES
75. What are different types of images?
Boiler plate imagesImage Items
76. What is the difference between boiler plat images and image items?
Boiler plate Images are static images (Either vector or bit map) that you import from the file system or database to use a grapical elements in your form, such as company logos and maps Image items are special types of interface controls that store and display either vector or bitmap images. Llike other items that store values, image items can be either base table items(items that relate directly to database columns) or control items. The definition of an image item is stored as part of the form module FMB and FMX files, but no image file is actualy associated with an image item until the item is populate at run time.
77. What are the trigger associated with image items?
When-image-activated fires when the operators double clicks on an image item when-image-pressed fires when an operator clicks or double clicks on an image item
78. What is the use of image_zoom built-in?
To manipulate images in image items.
WORKING WITH MULTIPLE FORMS
79. How do you create a new session while open a new form?
Using open_form built-in setting the session option Ex. Open_form('Stocks ',active,session). when invoke the mulitiple forms with open form and call_form in the same application, state whether the following are true/False
80. Any attempt to navigate programatically to disabled form in a call_form stack is allowed?
False
81. An open form can not be execute the call_form procedure if you chain of called forms has been initiated by another open form?
True
82. When a form is invoked with call_form, Does oracle forms issues a save point?
True
Mouse Operations
83. What are the various sub events a mouse double click event involves?
Double clicking the mouse consists of the mouse down, mouse up, mouse click, mouse down & mouse up events.
84, State any three mouse events system variables?
System.mouse_button_pressedSystem.mouse_button_shift_statesystem.mouse_itemsystem.mouse_canvassystem.mouse_record
OLE
85. What is an OLE?
Object Linking & Embadding provides you with the capability to integrate objects from many Ms-Windows applications into a single compound document creating integrated applications enables you to use the features form .
86. What is the difference between object embedding & linking in Oracle forms?
In Oracle forms, Embedded objects become part of the form module, and linked objects are references from a form module to a linked source file.
87. What is the difference between OLE Server & Ole Container?
An Ole server application creates ole Objects that are embedded or linked in ole Containers ex. Ole servers are ms_word & ms_excell. OLE containers provide a place to store, display and manipulate objects that are created by ole server applications. Ex. oracle forms is an example of an ole Container.
88. What are the different styles of actvation of ole Objects?
In place activationExternal activation
ViSUAL Attributes & property clauses
89. What are visual attributes?
Visual attributes are the font, color, pattern proprities that you set for form and menu objects that appear in your application interface.
90. What is a property clause?
A property clause is a named object that contains a list of properties and thier settings. Once you create a property clause you can base other object on it. An object based on a property can inherit the setting of any property in the clause that makes sense for that object.
91. Can a property clause itself be based on a property clause?
Yes
92. What are the important difference between property clause and visual attributes?
Named visual attributes differed only font, color & pattern attributes, property clauses can contain this and any other properties. You can change the appearance of objects at run time by changing the named visual attributes programatically , property clause assignments cannot be changed programatically. When an object is inheriting from both a property clause and named visual attribute, the named visual attribute settings take precedence, and any visual attribute properties in the class are ignored.
Form Build-ins
93. What is a Text_io Package?
It allows you to read and write information to a file in the file system.
94. What is an User_exit?
Calls the user exit named in the user_exit_string. Invokes a 3Gl programe by name which has been properly linked into your current oracle forms executable.
95. What is synchronize?
It is a terminal screen with the internal state of the form. It updates the screen display to reflect the information that oracle forms has in its internal representation of the screen.
96. What is forms_DDL?
Issues dynamic Sql statements at run time, including server side pl/SQl and DDL
Triggers
97. What is WHEN-Database-record trigger?
Fires when oracle forms first marks a record as an insert or an update. The trigger fires as soon as oracle forms determines through validation that the record should be processed by the next post or commit as an insert or update. c generally occurs only when the operators modifies the first item in the record, and after the operator attempts to navigate out of the item.
98. What are the master-detail triggers?
On-Check_delete_masterOn_clear_detailsOn_populate_details
99. What is the difference between $$DATE$$ & $$DBDATE$$
$$DBDATE$$ retrieves the current database date$$date$$ retrieves the current operating system date.
100. What is system.coordination_operation?
It represents the coordination causing event that occur on the master block in master-detail relation.
101. What are the difference between lov & list item?
Lov is a property where as list item ias an item. A list item can have only one column, lov can have one or more columns.
102. What are the different display styles of list items?
Pop_listText_listCombo box
103. What is pop list?
The pop list style list item appears initially as a single field (similar to a text item field). When the operator selects the list icon, a list of available choices appears.
104. What is a text list?
The text list style list item appears as a rectangular box which displays the fixed number of values. When the text list contains values that can not be displayed, a vertical scroll bar appears, allowing the operator to view and select undisplayed values.
105. What is a combo box?
A combo box style list item combines the features found in list and text item. Unlike the pop list or the text list style list items, the combo box style list item will both display fixed values and accept one operator entered value.
106. What are display items?
Display items are similar to text items with the exception that display items only store and display fetched or assigned values.Display items are generaly used as boilerplate or conditional text.
107. What is difference between open_form and call_form?
when one form invokes another form by executing open_form the first form remains displayed,and operators can navigate between the forms as desired. when one form invokes another form by executing call_form,the called form is modal with respect to the calling form.That is, any windows that belong to the calling form are disabled, and operators cannot navigate to them until they first exit the called form.
108. What is new_form built-in?
When one form invokes another form by executing new_form oracle form exits the first form and releases its memory before loading the new form calling new form completely replace the first with the second. If there are changes pending in the first form,the operator will be prompted to save them before the new form is loaded.
109. What is a library?
A library is a collection of subprograms including user named procedures, functions and packages.
110. What is the advantage of the library?
Library's provide a convenient means of storing client-side program units and sharing them among multipule applications. Once you create a library, you can attach it to any other form,menu,or library modules. When you can call library program units from triggers menu items commands and user named routine, you write in the modules to which you have attach the library.when a library attaches another library ,program units in the first library can reference program units in the attached library. Library support dynamic loading-that is library program units are loaded into an application only when needed. This can significantly reduce the run-time memory requirements of an applications.
111. What is strip sources generate options?
Removes the source code from the library file and generates a library files that contains only pcode.The resulting file can be used for final deployment, but can not be subsequently edited in the designer.
ex. f45gen module=old_lib.pll userid=scott/tigerstrip_source YES output_file
112.What are the vbx controls?
Vbx control provide a simple mehtod of buildig and enhancing user interfaces.The controls can use to obtain user inputs and display program outputs.vbx control where originally develop as extensions for the ms visual basic environments and include such items as sliders,grides and knobs.
113. What is a timer?
Timer is a "internal time clock" that you can programmatically create to perform an action each time the timer expires.
114. What are built-ins associated with timers?
find_timercreate_timerdelete_timer
115. what are difference between post database commit and post-form commit?
Post-form commit fires once during the post and commit transactions process, after the database commit occures. The post-form-commit trigger fires after inserts,updates and deletes have been posted to the database but before the transactions have been finalished in the issuing the command.The post-database-commit trigger fires after oracle forms issues the commit to finalished transactions.
116. What is a difference between pre-select and pre-query?
Fires during the execute query and count query processing after oracle forms constructs the select statement to be issued, but before the statement is actually issued.
The pre-query trigger fires just before oracle forms issues the select statement to the database after the operator as define the example records by entering the query criteria in enter query mode.
Pre-query trigger fires before pre-select trigger.
117. What is trigger associated with the timer?
When-timer-expired.
118 What is the use of transactional triggers?
Using transactional triggers we can control or modify the default functionality of the oracle forms.
REPORTS
1. What are the different file extensions that are created by oracle reports?
Rep file and Rdf file.
2. From which designation is it preferred to send the output to the printed?
Previewer.
3. Is it possible to disable the parameter from while running the report? Yes
4. What is lexical reference?How can it be created?
Lexical reference is place_holder for text that can be embedded in a sqlstatements.A lexical reference can be created using & before the column orparameter name.
5. What is bind reference and how can it carate?
Bind reference are used to replace the single value in sql,pl/sqlstatements a bind reference can be careated using a (:) before a column ora parameter name.
6.What use of command line parameter cmd file?
It is a command line argument that allows you to specify a file that contain a set of arguments for r20run.
7.Where is a procedure return in an external pl/sql library executed at the client or at the server?
At the client.
8. Where is the external query executed at the client or the server?
At the server.
9. What are the default parameter that appear at run time in the parameter screen?
Destype and Desname.
10. Which parameter can be used to set read level consistency across multiple queries?
Read only.
11. What is term?
The term is terminal definition file that describes the terminal form which you are using r20run.
12. What is use of term?
The term file which key is correspond to which oracle report functions.
13. Is it possible to insert comments into sql statements return in the data model editor?
Yes.
14. If the maximum record retrieved property of the query is set to 10 then a summary value will be calculated?
Only for 10 records.
15. What are the sql clauses supported in the link property sheet?
Where startwith having.
16. To execute row from being displayed that still use column in the row which property can be used?
Format trigger.
17. Is it possible to set a filter condition in a cross product group in matrix reports?
No.
18. If a break order is set on a column would it effect columns which are under the column? No.
19. With which function of summary item is the compute at options required?
percentage of total functions.
20. What is the purpose of the product order option in the column property sheet?
To specify the order of individual group evaluation in a cross products.
21.Can a formula column be obtained through a select statement?
Yes.
22.Can a formula column refered to columns in higher group?
Yes.
23. How can a break order be created on a column in an existing group?
By dragging the column outside the group.
24. What are the types of calculated columns available?
Summary, Formula, Placeholder column.
25. What is the use of place holder column?
A placeholder column is used to hold a calculated values at a specified place rather than allowing is to appear in the actual row where it has to appeared.
26. What is the use of hidden column?
A hidden column is used to when a column has to embedded into boilerplate text.
27. What is the use of break group?
A break group is used to display one record for one group ones.While multiple related records in other group can be displayed.
28. If two groups are not linked in the data model editor, What is the hierarchy between them?
Two group that is above are the left most rank higher than the group that is to right or below it.
29.The join defined by the default data link is an outer join yes or no?
Yes.
30. How can a text file be attached to a report while creating in the report writer?
By using the link file property in the layout boiler plate property sheet.
31. Can a repeating frame be careated without a data group as a base?
No.
32. Can a field be used in a report wihtout it appearing in any data group?
Yes.
33. For a field in a repeating frame, can the source come from the column which does not exist in the data group which forms the base for the frame?
Yes.
34. Is it possible to center an object horizontally in a repeating frame that has a variable horizontal size?
Yes.
35. If yes,how?
By the use anchors.
36. What are the two repeating frame always associated with matrix object?
One down repeating frame below one across repeating frame.
37. Is it possible to split the printpreviewer into more than one region?
Yes.
38. Does a grouping done for objects in the layout editor affect the grouping done in the datamodel editor?
No.
39. How can a square be drawn in the layout editor of the report writer?
By using the rectangle tool while pressing the (Constraint) key.
40. To display the page no. for each page on a report what would be the source & logical page no. or & of physical page no.?
& physical page no.
41. What does the term panel refer to with regard to pages?
A panel is the no. of physical pages needed to print one logical page.
42. What is an anchoring object & what is its use?
An anchoring object is a print condition object which used to explicitly or implicitly anchor other objects to itself.
43. What is a physical page ? & What is a logical page ?
A physical page is a size of a page. That is output by the printer. Thelogical page is the size of one page of the actual report as seen in thePreviewer.
44. What is the frame & repeating frame?
A frame is a holder for a group of fields. A repeating frame is used todisplay a set of records when the no. of records that are to displayed isnot known before.
REPORT TRIGGERS.
45. What are the triggers available in the reports?
Before report, Before form, After form , Between page, After report.
46. Does a Before form trigger fire when the parameter form is suppressed.
Yes.
47. At what point of report execution is the before Report trigger fired?
After the query is executed but before the report is executed and therecords are displayed.
48. Is the After report trigger fired if the report execution fails?
Yes.
49. Give the sequence of execution of the various report triggers?
Before form , After form , Before report, Between page, After report.
50. Is it possible to modify an external query in a report which containsit?
No.
51. What are the ways to monitor the performance of the report?
Use reports profile executable statement.Use SQL trace facility.
52. Why is it preferable to create a fewer no. of queries in the datamodel.
Because for each query, report has to open a separate cursor and has torebind, execute and fetch data.
53. What are the various methods of performing a calculation in a report ?
1. Perform the calculation in the SQL statements itself.2. Use a calculated / summary column in the data model.
54. Which of the above methods is the faster method?
performing the calculation in the query is faster.
55. Why is a Where clause faster than a group filter or a format trigger?
Because, in a where clause the condition is applied during data retrievalthan after retrieving the data.
56. What is the main diff. bet. Reports 2.0 & Reports 2.5?
Report 2.5 is object oriented.
57. What is the diff. bet. setting up of parameters in reports 2.0 reports2.5?
LOVs can be attached to parameters in the reports 2.5 parameter form.
58. How is link tool operation different bet. reports 2 & 2.5?
In Reports 2.0 the link tool has to be selected and then two fields to belinked are selected and the link is automatically created. In 2.5 the firstfield is selected and the link tool is then used to link the first field tothe second field.
REPORT 2.5 SPECIFIC ISSUES.
59.What are the two types views available in the object navigator(specificto report 2.5)?
View by structure and view by type .
60. Which of the two views should objects according to possession?
view by structure.
61.How is possible to restrict the user to a list of values while enteringvalues for parameters?
By setting the Restrict To List property to true in the parameter propertysheet.
62. How is it possible to select generate a select ste. for the query inthe query property sheet?
By using the tables/columns button and then specifying the table and thecolumn names.
63. If a parameter is used in a query without being previously defined,what diff. exist betw. report 2.0 and 2.5 when the query is applied?
While both reports 2.0 and 2.5 create the parameter, report 2.5 gives amessage that a bind parameter has been created.
64. Do user parameters appear in the data modal editor in 2.5?
No.
65.What is the diff. when confine mode is on and when it is off?
When confine mode is on, an object cannot be moved outside its parent inthe layout.
66. What is the diff. when Flex mode is mode on and when it is off?
When flex mode is on, reports automatically resizes the parent when thechild is resized.
67. How can a button be used in a report to give a drill down facility?
By setting the action asscoiated with button to Execute pl/sql option andusing the SRW.Run_report function.
68. What are the two ways by which data can be generated for a parameter'slist of values?
1. Using static values.2. Writing select statement.
69. What are the two panes that Appear in the design time pl/sqlinterpreter?
1.Source pane. 2. Interpreter pane
70. What are three panes that appear in the run time pl/sql interpreter?
1.Source pane. 2. interpreter pane. 3. Navigator pane.
CROSS PRODUCTS AND MATRIX REPORTS
71. How can a cross product be created?
By selecting the cross products tool and drawing a new group surroundingthe base group of the cross products.
72. How can a group in a cross products be visually distinguished from agroup that does not form a cross product?
A group that forms part of a cross product will have a thicker border.
73. Atleast how many set of data must a data model have before a data modelcan be base on it?
Four.
74. Is it possible to have a link from a group that is inside a crossproduct to a group outside ? (Y/N)
No.
75. Is it possible to link two groups inside a cross products after thecross products group has been created?
No.
76. What is an user exit used for?
A way in which to pass control (and possibly arguments ) form Oracle reportto another Oracle products of 3 GL and then return control ( and ) backto Oracle reprots.
77. What are the three types of user exits available ?
Oracle Precompiler exits, Oracle call interface,NonOracle user exits.
78. How can values be passed bet. precompiler exits & Oracle callinterface?
By using the statement EXECIAFGET & EXECIAFPUT.
79. How can I message to passed to the user from reports?
By using SRW.MESSAGE function.
Oracle DBA
1. SNAPSHOT is used for[DBA] a] Synonym, b] Table space, c] System server, d] Dynamic datareplication
Ans : D
2. We can create SNAPSHOTLOG for[DBA] a] Simple snapshots, b] Complex snapshots, c] Both A & B, d]Neither A nor B
Ans : A
3. Transactions per rollback segment is derived from[DBA] a] Db_Block_Buffers, b] Processes, c] Shared_Pool_Size, d] Noneof the above
Ans : B
4. ENQUEUE resources parameter information is derived from[DBA] a] Processes or DDL_LOCKS and DML_LOCKS, b] LOG_BUFFER,c] DB__BLOCK_SIZE..Ans : A
5. LGWR process writes information intoa] Database files, b] Control files, c] Redolog files, d] All theabove.Ans : C
6. SET TRANSACTION USE ROLLBACK SEGMENT is used to create userobjectsin a particular Tablespacea] True, b] FalseAns : False
7. Databases overall structure is maintained in a file calleda] Redolog file, b] Data file, c] Control file, d] All of theabove.Ans : C
8. These following parameters are optional in init.ora parameter fileDB_BLOCK_SIZE,PROCESSESa] True, b] FalseAns : False
9. Constraints cannot be exported through EXPORT commanda] True, b] FalseAns : False
10. It is very difficult to grant and manage common privileges needed bydifferent groups ofdatabase users using the rolesa] True, b] FalseAns : False
11. What is difference between a DIALOG WINDOW and a DOCUMENT WINDOWregardingmoving the window with respect to the application windowa] Both windows behave the same way as far as moving the window isconcerned.b] A document window can be moved outside the application window whilea dialogwindow cannot be movedc] A dialog window can be moved outside the application window while adocumentwindow cannot be movedAns : C
12. What is the difference between a MESSAGEBOX and an ALERTa] A messagebox can be used only by the system and cannot be used inuser applicationwhile an alert can be used in user application also.b] A alert can be used only by the system and cannot be use din userapplicationwhile an messagebox can be used in user application also.c] An alert requires an response from the userwhile a messagebox justflashes a messageand only requires an acknowledment from the userd] An message box requires an response from the userwhile a alert justflashes amessage an only requires an acknowledment from the userAns : C
13. Which of the following is not an reason for the fact that most of theprocessing is done at theserver ?a] To reduce network traffic. b] For application sharing, c] Toimplement business rulescentrally, d] None of the aboveAns : D
14. Can a DIALOG WINDOW have scroll bar attached to it ?a] Yes, b] NoAns : B
15. Which of the following is not an advantage of GUI systems ?a] Intuitive and easy to use., b] GUI's can display multipleapplications in multiple windowsc] GUI's provide more user interface objects for a developerd] None of the aboveAns :D
16. What is the difference between a LIST BOX and a COMBO BOX ?a] In the list box, the user is restricted to selecting a value from alist but in a combo boxthe user can type in a value which is not in the listb] A list box is a data entry area while a combo box can be used onlyfor control purposesc] In a combo box, the user is restricted to selecting a value from alist but in a list box theuser can type in a value which is not in the listd] None of the aboveAns : A
17. In a CLIENT/SERVER environment , which of the following would not bedone at the client ?a] User interface part, b] Data validation at entry line, c]Responding to user events,d] None of the aboveAns : D
18. Why is it better to use an INTEGRITY CONSTRAINT to validate data in atable than to use aSTORED PROCEDURE ?a] Because an integrity constraint is automatically checked while datais inserted into orupdated in a table while a stored procedure has to bespecifically invokedb] Because the stored procedure occupies more space in the databasethan a integrityconstraint definitionc] Because a stored procedure creates more network traffic than aintegrity constraintdefinitionAns : A
19. Which of the following is not an advantage of a client/server model ?a] A client/server model allows centralised control of data andcentralised implementationof business rules.b] A client/server model increases developer;s productivityc] A client/server model is suitable for all applicationsd] None of the above.Ans : C
20. What does DLL stands for ?a] Dynamic Language Libraryb] Dynamic Link Libraryc] Dynamic Load Libraryd] None of the aboveAns : B
21. POST-BLOCK trigger is aa] Navigational triggerb] Key triggerc] Transactional triggerd] None of the aboveAns : A
22. The system variable that records the select statement that SQL * FORMSmost recently usedto populate a block isa] SYSTEM.LAST_RECORDb] SYSTEM.CURSOR_RECORDc] SYSTEM.CURSOR_FIELDd] SYSTEM.LAST_QUERYAns: D
23. Which of the following is TRUE for the ENFORCE KEY fielda] ENFORCE KEY field characterstic indicates the source of the valuethat SQL*FORMSuses to populate the fieldb] A field with the ENFORCE KEY characterstic should have the INPUTALLOWEDcharaterstic turned offa] Only 1 is TRUEb] Only 2 is TRUEc] Both 1 and 2 are TRUEd] Both 1 and 2 are FALSEAns : A
24. What is the maximum size of the page ?a] Characters wide & 265 characters lengthb] Characters wide & 265 characters lengthc] Characters wide & 80 characters lengthd] None of the aboveAns : B
25. A FORM is madeup of which of the following objectsa] block, fields only,b] blocks, fields, pages only,c] blocks, fields, pages, triggers and form level procedures,d] Only blocks.Ans : C
26. For the following statements which is true1] Page is an object owned by a form2] Pages are a collection of display information such as constant textand graphics.a] Only 1 is TRUEb] Only 2 is TRUEc] Both 1 & 2 are TRUEd] Both are FALSEAns : B
27. The packaged procedure that makes data in form permanent in theDatabase isa] Postb] Post formc] Commit formd] None of the aboveAns : C
28. Which of the following is TRUE for the SYSTEM VARIABLE $$date$$a] Can be assigned to a global variableb] Can be assigned to any field only during design timec] Can be assigned to any variable or field during run timed] None of the aboveAns : B
29. Which of the following packaged procedure is UNRESTRICTED ?a] CALL_INPUT, b] CLEAR_BLOCK, c] EXECUTE_QUERY, d] USER_EXITAns : D
30. Identify the RESTRICTED packaged procedure from the followinga] USER_EXIT, b] MESSAGE, c] BREAK, d] EXIT_FORMAns : D
31. What is SQL*FORMSa] SQL*FORMS is a 4GL tool for developing & executing Oracle basedinteractiveapplications.b] SQL*FORMS is a 3GL tool for connecting to the Database.c] SQL*FORMS is a reporting toold] None of the above.Ans : A
32. Name the two files that are created when you generate a form usingForms 3.0a] FMB &amp;amp;amp; FMX, b] FMR & FDX, c] INP & FRM, d] None of the aboveAns : C
33. What is a triggera] A piece of logic written in PL/SQLb] Executed at the arrival of a SQL*FORMS eventc] Both A & Bd] None of the aboveAns : C
34. Which of the folowing is TRUE for a ERASE packaged procedure1] ERASE removes an indicated Global variable & releases the memoryassociated with it2] ERASE is used to remove a field from a page1] Only 1 is TRUE2] Only 2 is TRUE3] Both 1 & 2 are TRUE4] Both 1 & 2 are FALSEAns : 1
35. All datafiles related to a Tablespace are removed when the Tablespaceis droppeda] TRUEb] FALSEAns : B
36. Size of Tablespace can be increased bya] Increasing the size of one of the Datafilesb] Adding one or more Datafilesc] Cannot be increasedd] None of the aboveAns : B
37. Multiple Tablespaces can share a single datafilea] TRUEb] FALSEAns : B
38. A set of Dictionary tables are createda] Once for the Entire Databaseb] Every time a user is createdc] Every time a Tablespace is createdd] None of the aboveAns : A
39. Datadictionary can span across multiple Tablespacesa] TRUEb] FALSEAns : B
40. What is a DATABLOCKa] Set of Extentsb] Set of Segmentsc] Smallest Database storage unitd] None of the aboveAns : C
41. Can an Integrity Constraint be enforced on a table if some existingtable data does not satisfythe constrainta] Yesb] NoAns : B
42. A column defined as PRIMARY KEY can have NULL'sa] TRUEb] FALSEAns : B
43. A Transaction endsa] Only when it is Committedb] Only when it is Rolledbackc] When it is Committed or Rolledbackd] None of the aboveAns : C
44. A Database Procedure is stored in the Databasea] In compiled formb] As source codec] Both A & Bd] Not storedAns : C
45. A database trigger doesnot apply to data loaded before the definitionof the triggera] TRUEb] FALSEAns : A
46. Dedicated server configuration isa] One server process - Many user processesb] Many server processes - One user processc] One server process - One user processd] Many server processes - Many user processesAns : C
47. Which of the following does not affect the size of the SGAa] Database bufferb] Redolog bufferc] Stored procedured] Shared poolAns : C
48. What does a COMMIT statement do to a CURSORa] Open the Cursorb] Fetch the Cursorc] Close the Cursord] None of the aboveAns : D
49. Which of the following is TRUE1] Host variables are declared anywhere in the program2] Host variables are declared in the DECLARE sectiona] Only 1 is TRUEb] Only 2 is TRUEc] Both 1 & 2are TRUEd] Both are FALSEAns : B
50. Which of the following is NOT VALID is PL/SQLa] Bool boolean;b] NUM1, NUM2 number;c] deptname dept.dname%type;d] date1 date := sysdateAns : B
51. Declarefvar number := null; svar number := 5Begingoto <<>>if fvar is null then<<>>svar := svar + 5end if;End;
What will be the value of svar after the execution ?a] Errorb] 10c] 5d] None of the above
Ans : A
52. Which of the following is not correct about an Exception ?a] Raised automatically / Explicitly in response to an ORACLE_ERRORb] An exception will be raised when an error occurs in that blockc] Process terminates after completion of error sequence.d] A Procedure or Sequence of statements may be processed.
Ans : C
53. Which of the following is not correct about User_Defined Exceptions ?a] Must be declaredb] Must be raised explicitlyc] Raised automatically in response to an Oracle errord] None of the above
Ans : C
54. A Stored Procedure is aa] Sequence of SQL or PL/SQL statements to perform specific functionb] Stored in compiled form in the databasec] Can be called from all client environmetsd] All of the above
Ans : D
55. Which of the following statement is falsea] Any procedure can raise an error and return an user message anderror numberb] Error number ranging from 20000 to 20999 are reserved for userdefined messagesc] Oracle checks Uniqueness of User defined errorsd] Raise_Application_error is used for raising an user defined error.
Ans : C
56. Is it possible to open a cursor which is in a Package in anotherprocedure ?a] Yesb] No
Ans : A
57. Is it possible to use Transactional control statements in DatabaseTriggers ?a] Yesb] No
Ans : B
58. Is it possible to Enable or Disable a Database trigger ?a] Yesb] No
Ans : A
59. PL/SQL supports datatype(s)a] Scalar datatypeb] Composite datatypec] All of the aboved] None of the above
Ans C
60. Find the ODD datatype outa] VARCHAR2b] RECORDc] BOOLEANd] RAW
Ans : B
61. Which of the following is not correct about the "TABLE" datatype ?a] Can contain any no of columnsb] Simulates a One-dimensional array of unlimited sizec] Column datatype of any Scalar typed] None of the above
Ans : A
62. Find the ODD one out of the followinga] OPENb] CLOSEc] INSERTd] FETCH
Ans C
63. Which of the following is not correct about Cursor ?a] Cursor is a named Private SQL areab] Cursor holds temporary resultsc] Cursor is used for retrieving multiple rowsd] SQL uses implicit Cursors to retrieve rows
Ans : B
64. Which of the following is NOT VALID in PL/SQL ?a] Select ... intob] Updatec] Created] Delete
Ans : C
65. What is the Result of the following 'VIK'NULL'RAM' ?a] Errorb] VIK RAMc] VIKRAMd] NULL
Ans : C
66. Declarea number := 5; b number := null; c number := 10;Beginif a > b AND a <> ( Select count(*) from Emp E2where E1.SAL > E2.SAL ) will retrievea] 3500,5000,2500b] 5000,2850c] 2850,5750d] 5000,5750
Ans : A
72. Is it possible to modify a Datatype of a column when column containsdata ?a] Yesb] No
Ans B
73. Which of the following is not correct about a View ?a] To protect some of the columns of a table from other usersb] Ocuupies data storage spacec] To hide complexity of a queryd] To hide complexity of a calculations
Ans : B
74. Which is not part of the Data Definiton Language ?a] CREATEb] ALTERc] ALTER SESSION
Ans : C
75. The Data Manipulation Language statements area] INSERTb] UPDATEc] SELECTd] All of the above
Ans : D
76. EMPNO ENAME SALA822 RAMASWAMY 3500A812 NARAYAN 5000A973 UMESHA500 BALAJI 5750
Using the above dataSelect count(sal) from Emp will retrievea] 1b] 0c] 3d] None of the above
Ans : C
77. If an UNIQUE KEY constraint on DATE column is created, will it acceptthe rows that areinserted with SYSDATE ?a] Willb] Won't
Ans : B
78. What are the different events in Triggers ?a] Define, Createb] Drop, Commentc] Insert, Update, Deleted] All of the above
Ans : C79. What built-in subprogram is used to manipulate images in image items ?a] Zoom_outb] Zoom_in'c] Image_zoomd] Zoom_image
Ans : C
80. Can we pass RECORD GROUP between FORMS ?a] Yesb] No
Ans : A
81. SHOW_ALERT function returnsa] Booleanb] Numberc] Characterd] None of the above
Ans : B
82. What SYSTEM VARIABLE is used to refer DATABASE TIME ?a] $$dbtime$$b] $$time$$c] $$datetime$$d] None of the above
Ans : A
83. :SYSTEM.EFFECTIVE.DATE varaible isa] Read onlyb] Read & Writec] Write onlyd] None of the above
Ans : C
84. How can you CALL Reports from Forms4.0 ?a] Run_Report built_inb] Call_Report built_inc] Run_Product built_ind] Call_Product built_in
Ans : C
85. When do you get a .PLL extension ?a] Save Library fileb] Generate Library filec] Run Library filed] None of the above
Ans : A
86. What is built_in Subprogram ?a] Stored procedure & Functionb] Collection of Subprogramc] Collection of Packagesd] None of the above
Ans : D
87. GET_BLOCK property is aa] Restricted procedureb] Unrestricted procedurec] Library functiond] None of the above
Ans : D
88. A CONTROL BLOCK can sometimes refer to a BASETABLE ?a] TRUEb] FALSE
Ans : B
89. What do you mean by CHECK BOX ?a] Two state controlb] One state controlc] Three state controld] none of the above
Ans : C - Please check the Correcness of this Answer ( The correct answeris 2 )
90. List of Values (LOV) supportsa] Single columnb] Multi columnc] Single or Multi columnd] None of the above
Ans : C
91. What is Library in Forms 4.0 ?a] Collection of External fieldb] Collection of built_in packagesc] Collection of PL/SQl functions, procedures and packagesd] Collection of PL/SQL procedures & triggers
Ans : C
92. Can we use a RESTRICTED packaged procedure in WHEN_TEXT_ITEM trigger ?a] Yesb] No
Ans : B
93. Can we use GO_BLOCK package in a PRE_TEXT_ITEM trigger ?a] Yesb] No
Ans : B
94. What type of file is used for porting Forms 4.5 applications to variousplatforms ?a] .FMB fileb] .FMX filec] .FMT filed] .EXE file
Ans : C
95. What built_in procedure is used to get IMAGES in Forms 4.5 ?a] READ_IMAGE_FILEb] GET_IMAGE_FILEc] READ_FILEd] GET_FILE
Ans A
96. When a form is invoked with CALL_FORM does Oracle forms issuesSAVEPOINT ?a] Yesb] No
Ans : A
97. Can we attach the same LOV to different fields in Design time ?a] Yesb] No
Ans : A
98. How do you pass values from one form to another form ?a] LOVb] Parametersc] Local variablesd] None of the above
Ans : B
99. Can you copy the PROGRAM UNIT into an Object group ?a] Yesb] No
Ans : B
100. Can MULTIPLE DOCUMENT INTERFACE (MDI) be used in Forms 4.5 ?a] Yesb] No
Ans : A
101. When is a .FMB file extension is created in Forms 4.5 ?a] Generating formb] Executing formc] Save formd] Run form
Ans : C
102. What is a Built_in subprogram ?a] Libraryb] Stored procedure & Functionc] Collection of Subprogramsd] None of the above
Ans : D
103. What is a RADIO GROUP ?a] Mutually exclusiveb] Select more than one columnc] Above all TRUEd] Above all FALSE
Ans : A
104. Identify the Odd one of the following statements ?a] Poplistb] Tlistc] List of valuesd] Combo box
Ans : C
105. What is an ALERT ?a] Modeless windowb] Modal windowc] Both are TRUEd] None of the above
Ans : B
106. Can an Alert message be changed at runtime ?a] Yesb] No
Ans : A
107. Can we create an LOV without an RECORD GROUP ?a} Yesb] No
Ans : B
108. How many no of columns can a RECORD GROUP have ?a] 10b] 20c] 50d] None of the above
Ans D
109. Oracle precompiler translates the EMBEDDED SQL statemens intoa] Oracle FORMSb] Oracle REPORTSc] Oracle LIBRARYd] None of the above
Ans : D
110. Kind of COMMENT statements placed within SQL statements ?a] Asterisk(*) in column ?b] ANSI SQL style statements(...)c] C-Style comments (/*......*/)d] All the above
Ans : D
111. What is the appropriate destination type to send the output to aprinter ?a] Screenb] Previewerc] Either of the aboved] None of the above
Ans : D
112. What is TERM ?a] TERM is the terminal definition file that describes the terminalfrom which you areusing R20RUN ( Reports run time )b] TERM is the terminal definition file that describes the terminalfrom which you areusing R20DES ( Reports designer )c] There is no Parameter called TERM in Reports 2.0d] None of the above
Ans : A
113. If the maximum records retrieved property of a query is set to 10,then a summary value willbe calculateda] Only for 10 recordsb] For all the records retrievedc] For all therecords in the referenced tabled] None of the above
Ans : A
114. With which function of a summary item in the COMPUTE AT optionrequired ?a] Sumb] Standard deviationc] Varianced] % of Total function
Ans : D
115. For a field in a repeating frame, can the source come from a columnwhich does not exist inthe datagroup which forms the base of the frame ?a] Yesb] No
Ans : A
116. What are the different file extensions that are created by OracleReports ?a] .RDF file & .RPX fileb] .RDX file & .RDF filec] .REP file & .RDF filed] None of the above
Ans : C
117. Is it possible to Disable the Parameter form while running the report?a] Yesb] No
Ans : A
118.What are the SQL clauses supported in the link property sheet ?a] WHERE & START WITHb] WHERE & HAVINGc} START WITH & HAVINGd] WHERE, START WITH & HAVING
Ans : D
119. What are the types of Calculated columns available ?a] Summary, Place holder & Procedure columnb] Summary, Procedure & Formula columnsc] Procedure, Formula & Place holder columnsd] Summary, Formula & Place holder columns
Ans.: D
120. If two groups are not linked in the data model editor, what is thehierarchy between them?a] There is no hierarchy between unlinked groupsb] The group that is right ranks higher than the group that is to theleftc] The group that is above or leftmost ranks higher than the groupthat is to right or belowitd] None of the above
Ans : C
121. Sequence of events takes place while starting a Database isa] Database opened, File mounted, Instance startedb] Instance started, Database mounted & Database openedc] Database opened, Instance started & file mountedd] Files mounted, Instance started & Database opened
Ans : B
122. SYSTEM TABLESPACE can be made off-linea] Yesb] No
Ans : B
123. ENQUEUE_RESOURCES parameter information is derived froma] PROCESS or DDL_LOCKS & DML_LOCKSb] LOG BUFFERc] DB_BLOCK_SIZEd] DB_BLOCK_BUFFERS
Ans : A
124. SMON process is used to write into LOG filesa] TRUEb] FALSE
Ans : B
125. EXP command is useda] To take Backup of the Oracle Databaseb] To import data from the exported dump filec] To create Rollback segmentsd] None of the above
Ans : A
126. SNAPSHOTS cannot be refreshed automaticallya] TRUEb] FALSEAns : B127. The User can set Archive file name formatsa] TRUEb] FALSE
Ans : A
128. The following parameters are optional in init.ora parameter fileDB_BLOCK_SIZE,PROCESSa} TRUEb] FALSE
Ans : B129. NOARCHIEVELOG parameter is used to enable the database in Archievemodea] TRUEb] FALSE
Ans : B
130. Constraints cannot be exported through Export command?a] TRUEb] FALSE
Ans : B
131. It is very difficult to grant and manage common priveleges needed bydifferent groups ofdatabase users using rolesa] TRUEb] FALSE
Ans : B
132. The status of the Rollback segment can be viewed througha] DBA_SEGMENTSb] DBA_ROLESc] DBA_FREE_SPACESd] DBA_ROLLBACK_SEG
Ans : D
133. Explicitly we can assign transaction to a rollback segmenta] TRUEB] FALSE
Ans : A
134. What file is read by ODBC to load drivers ?a] ODBC.INIb] ODBC.DLLc] ODBCDRV.INId] None of the above
Ans : A

What is normalization? - Well a relational database is basically composed of tables that contain related data. So the Process of organizing this data into tables is actually referred to as normalization.
What is a Stored Procedure? - Its nothing but a set of T-SQL statements combined to perform a single task of several tasks. Its basically like a Macro so when you invoke the Stored procedure, you actually run a set of statements.
Can you give an example of Stored Procedure? - sp_helpdb , sp_who2, sp_renamedb are a set of system defined stored procedures. We can also have user defined stored procedures which can be called in similar way.
What is a trigger? - Triggers are basically used to implement business rules. Triggers is also similar to stored procedures. The difference is that it can be activated when data is added or edited or deleted from a table in a database.
What is a view? - If we have several tables in a db and we want to view only specific columns from specific tables we can go for views. It would also suffice the needs of security some times allowing specfic users to see only specific columns based on the permission that we can configure on the view. Views also reduce the effort that is required for writing queries to access specific columns every time.
What is an Index? - When queries are run against a db, an index on that db basically helps in the way the data is sorted to process the query for faster and data retrievals are much faster when we have an index.
What are the types of indexes available with SQL Server? - There are basically two types of indexes that we use with the SQL Server. Clustered and the Non-Clustered.
What is the basic difference between clustered and a non-clustered index? - The difference is that, Clustered index is unique for any given table and we can have only one clustered index on a table. The leaf level of a clustered index is the actual data and the data is resorted in case of clustered index. Whereas in case of non-clustered index the leaf level is actually a pointer to the data in rows so we can have as many non-clustered indexes as we can on the db.
What are cursors? - Well cursors help us to do an operation on a set of data that we retrieve by commands such as Select columns from table. For example : If we have duplicate records in a table we can remove it by declaring a cursor which would check the records during retrieval one by one and remove rows which have duplicate values.
Can you tell me the difference between DELETE & TRUNCATE commands? - Delete command removes the rows from a table based on the condition that we provide with a WHERE clause. Truncate will actually remove all the rows from a table and there will be no data in the table after we run the truncate command.
Can we use Truncate command on a table which is referenced by FOREIGN KEY? - No. We cannot use Truncate command on a table with Foreign Key because of referential integrity.
What is the difference between a HAVING CLAUSE and a WHERE CLAUSE? - Having Clause is basically used only with the GROUP BY function in a query. WHERE Clause is applied to each row before they are part of the GROUP BY function in a query.
What do you mean by COLLATION? - Collation is basically the sort order. There are three types of sort order Dictionary case sensitive, Dictonary - case insensitive and Binary.